SlideShare a Scribd company logo
QA – 1 ]       || Ohm Sai Ram ||   [ Practice Paper



                  FM
 Chandresh Agrawal's
           bestsellers
Practice Paper
    Quantitative Aptitude
                 &
           Numerical Ability


www.clhandreshagrawalbooks.com
Practice Paper (Solved) ]                                                 [ QA – 2

                             Practice Paper (Solved)
                     QUANTITATIVE APTITUDE
                                                        3                  3
1.   Three students try to solve a
     problem independently with a               (a) k = 2          (b) k ≠ 2
     probability of solving it as
                                                (c) k ≠ 6          (d) k = 6
     1 2 5
       , ,     respectively. What is       5.   A bag contains 5 red balls and 8
     3 5 12
                                                blue balls. It also contains 4
     the probability that the
                                                green and 7 black balls. If a ball
     problem is solved ?
                                                is drawn at randomly find the
         23                                     probability that it is not green
                              7
     (a) 30            (b)                          5                1
                             30                     6
         32                30                   (a)              (b) 4

     (c) 30            (d) 7                        1                  7
                                                (c) 6              (d) 4
2.   From among 36 teachers in a
     school one principal and one
     vice principal are to be              6.   The letters of the word
     appointed. In how many ways                PROMISE are arranged so that
     can this be done ?                         no two of the vowels should
                                                come together. Find total
     (a) 1260          (b) 1250                 number of arrangements.
     (c) 1240          (d) 1800                 (a) 49             (b) 1440
3.   A boy has 3 library tickets and            (c) 7              (d) 1898
     8 books of his interest in the
     library. Of these 8, he does not      7.   Surendra,        Rajendra     and
     want to borrow chemistry part              Manindra        invested    some
     II, unless chemistry part I is             amount in a business in the
     also borrowed. In how many                 ratio of 5 :7 : 6 respectively. In
     ways can he choose the three               the next year they increased
     books to be borrowed ?                     their investments by 26%, 20%
                                                and 15% respectively. The profit
     (a) 56            (b) 27                   earned during the second year
     (c) 26            (d) 41                   should be distributed in what
                                                ratio     among       Surendra,
4.   If the system 2x + 3y − 5 = 0, 4x +        Rajendra       and    Manindra
     ky − 10 = 0 has an infinite                respectively ?
     number of solutions, then
                                                (a) 31 : 27 : 21   (b) 21 : 28 : 23
QA – 3 ]                                                          [ Practice Paper
     (c) 26 : 20 : 15                          (c) 564              (d) 645
     (d) Cannot be determined                  (e) None of these
     (e) None of these                     11. Which    of   the    following
                                               expression are exactly equal in
8.   Four of the following five parts
                                               value ?
     numbered (i), (ii), (iii), (iv) and
     (v) are exactly equal. The                I. (3x − y)2 − (5x2 − 2xy)
     number of the part which is not
                                               II. (2x − y)2
     equal to the remaining four is
     your answers.                             III. (2x + y)2 − 2xy
     (i) 36 × 15 ÷ 27 × 13                     IV. (2x + 3y)2 − 8y(2x + y)

     (ii) 53 × 4 × 64 ÷ 16 × 7                 (a) I and II only

     (iii) 328 ÷ 41 × 21 + 9 × 23              (b) I, II and III only
                                               (c) II and IV only
     (iv)      1024 × 11 − 16 × 7
                                               (d) I, II and IV only
     (v) 17 × 18 −      121 × 6                (e) None of these
     (a) I               (b) II            12. Salary of an officer increases
     (c) III             (d) IV                every year by 20%. His salary in
                                               the year 2001 was Rs. 26,640.
     (e) V
                                               What was his salary in 1999?
9.   A shopkeeper sold an article for
                                               (a) Rs. 20,000       (b) Rs. 19,028
     Rs. 6,750 after given a discount
     of 10% on the labelled price. He          (c) Rs. 18,500       (d) Rs. 18,840
     would have earned a profit of             (e) None of these
     50%, had there been no
     discount. What was the actual         13. What    approximate  value
     percentage of profit earned ?             should come in place of the
                                               question mark (?) in the
     (a) 36              (b) 40                following equation ?
     (c) 35                                    95.9753.5 ÷ 16.0013.5 × 6.0021.5 ÷
     (d) Cannot be determined                  35.992 = ?
     (e) None of these                         (a) 36               (b) 16
10. From a group of 7 men and 6                (c) 96               (d) 32
    women 5 persons are to be                  (e) 6
    selected to form a committee so
    that at least 3 men are there on       14. Mr. Anand deposited a total
    the committee. In how many                 amount of Rs. 65,000 in three
    different ways can it be done ?            different schemes A, B and C
                                               with rates of interest 12 p.c.p.a.,
     (a) 756             (b) 735               16 p.c.p.a. and 18 p.c.p.a.
Practice Paper (Solved) ]                                                   [ QA – 4
    respectively and earned a total         (a) 81 litres            (b) 72 litres
    interest of Rs. 10,180 in one
                                            (c) 54 litres            (d) 66 litres
    year. If the amount invested in
    Scheme A was 72% of the                        1              1
                                        18. If p +             p−
                                                2
    amount invested in Scheme 'C',                  2 = a, and      = b then
                                                  p               p
    what was the amount invested
                                            which of the following is
    in Scheme B ?
                                            correctly expressed ?
    (a) Rs. 25,000
                                            (a) a − b2 − 2 = 0
    (b) Rs. 22,000
                                            (b) a2 + b = 2
    (c) Rs. 18,000
                                            (c) a2 − b2 = 1          (d) a2 = b2
    (d) Cannot be determined
                                        19. If a + b + c = 0, then the value of
    (e) None of these
                                            a(c + a) (a + b) − b(a + b)(b + c) is
15. In how many different ways can          equal to :
    the letters of the word TRAINER
    be arranged so that the vowels          (a) 1                    (b) a b c
                                                a       b
    always come together?                           +
                                                    c
                                            (c)                      (d) 0
    (a) 1440            (b) 120
    (c) 720             (d) 360         20. A worker earns a 5% raise. A
                                            year later, the worker receives
    (e) None of these                       a 2.5% cut in pay, and now his
16. What will be the value of :             salary is Rs. 22702.68. What was
                                            his salary to begin with ?
      98 − 72 + 50
                                            (a) Rs. 22000            (b) Rs. 22176
              18
                                38          (c) Rs. 25000            (d) Rs. 22193
    (a) 6               (b)
          4                     3       21. What will come in place of the
    (c) − 3             (d) 2               question mark (?) in the
                                            following series ?
17. In a hotel, there are dishonest
    waiters. One of them takes out                2         3   10   39      172     ?
    one third of the milk from a            (a) 704                  (b) 885
    container full of milk and
    replaces it with equal quantity         (c) 785                  (d) 804
    of water. A second waiter again         (e) None of these
    takes out one third of the
                                        22. A man received a cheque in
    mixture and replaces it with
                                            which     the   rupees    were
    equal quantity of water. The
                                            transposed for paise and vice
    process is repeated by 4 waiters
                                            versa. After spending 5 rupees
    resulting in only 16 litres of
                                            42 paise, he discovered that he
    milk being left in the container.
                                            now had exactly six times the
    What is the capacity of the
                                            value of the correct cheque
    container ?
QA – 5 ]                                                     [ Practice Paper
    amount. What amount should           27. Monthly incomes of two
    he have received ?                       persons are in the ratio 5 : 4 and
                                             their monthly expenditures are
    (a) Rs. 6.44          (b) Rs. 3.22
                                             in the ratio of 9 : 7. If each
    (c) Rs. 18.25         (d) Rs. 8.36       person saves Rs. 500 per month,
23. If α and β are the roots of the          then what are their monthly
    quadratic equation ax2 + bx + c          incomes ?
    = 0, then the value of                   (a) Rs. 8000 and Rs. 10000
      2      2
     α    β                                  (b) Rs. 3750 and Rs. 3000
        +     is
     β    α
        3bc a                 3abc b         (c) Rs. 5000 and Rs. 4000
            − 3                    − 3       (d) None of these
    (a)    b c            (b)    a c
             2                    2
                                         28. Five persons A, B, C, D and E
        3abc b               ab b
             − 2               − 2c          occupy seats in a row such that
    (c)    a c            (d) 2b 2 c         A and B sit next to each other.
            3
                                             In how many possible ways can
24. 3 chairs and 2 tables cost Rs.           these five people sit ?
    700, while 5 chairs and 3 tables         (a) 24              (b) 48
    cost Rs. 1100. What is the cost
    of 2 chairs and 2 tables ?               (c) 72              (d) None of these

    (a) Rs. 300           (b) Rs. 350    29. 26   × 12 ÷ 8 + ? = 76
    (c) Rs. 450           (d) Rs. 600        (a) 39              (b) 42
25. If a, b are the two roots of a           (c) 43              (d) 37
    quadratic equation such that             (e) None of these
    a + b = 24 and a − b = 8, then the
    quadratic equation having a          30. The MSEB electricity bills are
    and b as its roots is                    calculated in the following
                                             manner. The change in meter
    (a) x2 + 2x + 8 = 0                      reading for the month is
    (b) x2 − 4x + 8 = 0                      rounded off to the next highest
                                             multiple of 10. The result is
    (c) x2 − 24x + 128 = 0                   multiplied by 55 paise, and the
    (d) 2x2 + 8x + 9 = 0                     sum is rounded off to the next
                                             rupee. If the reading last month
26. A sum of money was divided
                                             was 17385 units and this month
    among two persons x and y in
                                             it is 18293 units, what is the bill
    the ratio 4 : 5. x received Rs. 5
                                             for this month ?
    less than y. What is the total
    amount of money :                        (a) Rs. 501         (b) Rs. 495
    (a) 45                (b) 50             (c) Rs. 500         (d) Rs. 505
    (c) 90                (d) 250        31. A gambler pays Rs. 3 and gets
                                             to throw a dice. He receives an
Practice Paper (Solved) ]                                              [ QA – 6
    amount equal to the number               (c) 100 litres    (d) 120 litres
    that the top face of the dice
                                         37. There are 10 pairs of socks in a
    shows. If the gambler keeps on
                                             drawer. What is the minimum
    playing the game, how much
                                             number of socks that a person
    does he win per throw, in the
                                             should pull out from the drawer
    long run ?
                                             ensure that he gets at least 2
    (a) 50 Ps.          (b) Rs. 1            matching pairs of socks ?
    (c) −50 Ps.         (d) Rs. 0            (a) 12            (b) 11
      −              −
32. |P−10| = 12 & |4J− 10| = 6. What         (c) 5             (d) 10
                                 P       38. A painting show drew crowds
    is maximum value of            .         which doubled in number each
                                 J
                                             day. If the show opened on
    (a) −11             (b) 22               Monday and the number of
                            11
                                             spectators on Saturday was
    (c) −2              (d) 2                6400, what was the number on
33. If x + y > 5 and x − y > 3, then         the opening day?
    which of the following gives all         (a) 100           (b) 200
    possible values of x ?
                                             (c) 800           (d) 80
    (a) x > 3           (b) x > 4
                                         39. 15 chairs and 2 tables cost Rs.
    (c) x > 5           (d) x < 5            4,000. Find the cost of 12 chairs
34. 136 × 25 ÷ 16 × ? = 2550                 and 2 tables, if the cost of 10
                                             chairs be equal to that of 5
    (a) 12              (b) 22               tables.
    (c) 20              (d) 18               (a) Rs. 4,000     (b) Rs. 4,200
    (e) None of these                        (c) Rs. 3,900     (d) Rs. 3,600
35. The probability of rain on day       40. 5 chairs and 2 tables cost Rs.
    1 is 0.2 and the probability on          1,080. The cost of 2 chairs is
    day 2 is 0.3. What is the                equal to that of a table. Find the
    probability of raining on both           cost of 2 chairs and 5 tables.
    the days?
                                             (a) Rs. 1,440     (b) Rs. 1,480
    (a) 0.2             (b) 0.1
                                             (c) Rs. 1,380     (d) Rs. 1,420
    (c) 0.06            (d) 0.25
                                         41. An employee spends 30% of his
36. Three − fourths of a tank is full        salary on food and donates 3%
    of water. If 5 litres are added to       of his salary. If he spends Rs.
                  −
    it then four−fifths of the tank          231 on these two items what is
    becomes full. What is the                his salary ?
    capacity of the tank?
                                             (a) Rs. 1250      (b) Rs. 700
    (a) 75 litres       (b) 80 litres
                                             (c) Rs. 630       (d) Rs. 940
QA – 7 ]                                                             [ Practice Paper
                                                    23                     1
42. The mean annual salary paid to
    all the staff members of a                  (c) 30                 (d) 2
    company was Rs. 5000. The
                                            47. A student on his birthday
    mean annual salary paid to
                                                distributed on an average 5
    male and female staff were Rs.
                                                chocolates per student. If on the
    5200 and Rs. 4500 respectively.
                                                arrival of the teacher and the
    Determine the number of male
                                                headmaster to whom the
    and female staff members of the
                                                student gives 10 and 15
    company.
                                                chocolates respectively, the
    (a) 80, 20         (b) 70, 30               average chocolate distributed
    (c) 60, 40         (d) 40, 60               per head increases to 5.5, then
                                                what is the strength of the
43. A man has some hens & cows.                 class?
    If the number of heads be 48 and
    the number of feet equals 140,              (a) 28                 (b) 30
    the number of hens will be                  (c) 32                 (d) None of these
    (a) 26             (b) 24               48. Find the value of
    (c) 23             (d) 22                   0.2 × 0.2 × 0.2 + 0.02 × 0.02 × 0.02
                     2                          0.4 × 0.4 × 0.4 + 0.04 × 0.04 × 0.04
44. If   a b = 5b + a , then (a, b) could
    be                                                         0.67 × 0.67 × 0.67 − 0.001
                                                           +
                                                               0.67 × 0.67 + 0.067 + 0.01
    (a) (3, 4)         (b) (2, 12)
                                                (a) 4.87               (b) 1.07
    (c) (4, 18)        (d) (6, 4)
                                                (c) 0.067              (d) 0.002
45. How many bricks are required
    to build a wall of 15 metres            49. Find the value of
    length, 12 metres height and 20
                                                0.03 × 0.03 + 0.01 × 0.01 - 0.02 × 0.03
    cm thickness, if the brick is 36
    cm long, 25 cm wide and 10 cm                                0.02
    thick ?                                                                  FG 3 + 1 of 3 IJ
    (a) 2000           (b) 4000
                                                                         ÷
                                                                              H 8 2 16K
    (c) 12000          (d) None of these        (a) 0.447              (b) 8.04

46. Three students try to solve a               (c) 0.0427             (d) 0.012
    problem independently with a
    probability of solving it as                                  0.289         24 + 216
                                            50. Simplify :               +
     2 4 3                                                       0.00121            96
      ,    , respectively. What is
     8 5 4                                      (a) 12.45              (b) 16.54
    the probability that the
    problem is solved ?                         (c) 18.90              (d) 17.45
                        12
         3                                  51. A pack of 52 cards is distributed
    (a)             (b) 30                      amongst 4 players. The one to
        20
Practice Paper (Solved) ]                                                         [ QA – 8
    receive the set with the lowest        (c) 9                         (d) 12
    sum wins
                                       56. If   x1/3   +   y1/3   +   z1/3   = 0, then :
    (A = 1, J = 11, Q = 12, K = 13).
                                           (a) x + y + z = 0
    What is the least total with
                                           (b) (x + y + z)3 = 27 xyz
    which one can win ?
                                           (c) x + y + z = 3 xyz
    (a) 40             (b) 31
                                           (d) x3 + y3 + z3 = 0
    (c) 28             (d) 24
                                       57. A crown, made of gold, silver,
52. If every 2 out of 3 readymade
                                           copper and brass weighs 9.725
    shirts need alterations in the
                                           kg. The weight of the gold and
    collar, every 3 out of 4 need
                                           silver together is 4 kg and the
    alterations in the sleeves, and
                                           weight of the gold and copper
    every 4 out of 5 need it in the
                                           4.5 kg and of the gold and brass
    body, how many alterations
                                           3.6 kg. What is the weight of
    will be required for 60 shirts ?
                                           gold in the crown ?
    (a) 123            (b) 133
                                           (a) 1.2500 kg                 (b) 2.6575 kg
    (c) 143            (d) 24
                                           (c) 1.1875 kg                 (d) 2.3705 kg
53. Which of the following does not
                                       58. A bag contains 3 white balls
    belong to the group ?
                                           and 2 black balls. Another bag
    10 , 11 , 18, 36, 74                   contains 2 white balls and 4
                                           black balls. A bag and a ball are
    (a) 10             (b) 11
                                           picked     at   random.      The
    (c) 18             (d) 36              probability that the ball will be
54. Tulsi had a children's party and       white is
    bought two mangoes for each                 7               7
    child. However, a quarter of the       (a) 11           (b) 30
    kids invited did not come. 25
    boys came and the surplus                   5                             7
    provided just one extra mango          (c) 11                        (d) 15
    for each girl. How many
    mangoes did Tulsi buy ?            59. One hundred identical coins
    (a) 200            (b) 132             each with probability p of
                                           showing up heads are tossed. If
    (c) 150            (d) 128             0 < p < 1 and the probability of
                   −
55. If x is a three−digit number and       heads showing on 50 coins is
    y is a number obtained by              equal to that of heads on 51
    permuting the digits of x in any       coins, then the value of p is
    manner, then (x − y) is always             1                 49
    divisible by :                         (a) 2            (b) 101
    (a) 4              (b) 6
QA – 9 ]                                                      [ Practice Paper
         50                   51           65. With the same data as for the
    (c) 101              (d) 101               previous question, what is the
                                               probability that the next
60. Two dice are tossed. The                   program will run correctly
    probability that the total score           after the third run, but not
    is a prime number is                       earlier ?
                                                    9               3
        1                 5
                                               (a) 10          (b) 10
    (a) 6            (b) 12
                                                   7                    1
        1                    7                 (c) 20              (d) 10
    (c) 2                (d) 9
                                           66. A number is greater than the
                   FG 1 IJ   −2                square of 44 but smaller than
61. The value of
                    H 4K       1
                                  is           the square of 45. If one part of
                                               the number is the square of 6
                                               and the number is the multiple
    (a) 2                (b) − 2
                                               of 5, then find the number.
           1
                                               (a) 1940            (b) 2080
    (c) − 16             (d) 16
                                               (c) 1980
62. The volume of a cube is V. The
    total length of its edges is               (d) Cannot be determined
                                               (e) None of these
    (a) 6 V1/3           (b) 8         V
                                           67. Rajesh solved 80 per cent of the
    (c) 12 V2/3          (d) 12 V1/3           questions in an examination. If
63. A boy was asked to write 25 ×              out of 41 questions solved by
    9 2 , but he wrote 2592. The               Rajesh 37 questions are correct
    numerical difference between               and of the remaining questions
    the two is                                 out of 8 questions 5 questions
                                               have been solved by Rajesh
    (a) 0                (b) 1                 correctly then find the total
    (c) 2                (d) 3                 number of questions asked in
                                               the examination ?
            x 2 − 36         x+6
64. If P =           and Q =      , then       (a) 75              (b) 65
            x − 49
              2
                             x+7
                     P                         (c) 60
    the value of       is
                     Q                         (d) Cannot be determined
        x 6                 x 6
          −7                  −                (e) None of these
    (a) x               (b) x 7
          −                   +            68. What was the day on April 6,
        x                                      2001?
    (c) x −76               x+6
                                               (a) Friday          (b) Saturday
          +             (d) x − 7
                                               (c) Thursday        (d) Sunday
Practice Paper (Solved) ]                                            [ QA – 10
69. Monthly incomes of two               74. 76.59 + 129.052 − 38.314 = ? +
    persons are in the ratio 5 : 4 and       45.72
    their monthly expenditures are
    in the ratio of 9 : 7. If each           (a) 121.068         (b) 121.608
    person saves Rs. 500 per month,          (c) 120.068         (d) 120.608
    then what are their monthly
    incomes ?                                (e) None of these

    (a) Rs. 8000 and Rs. 10000           75. 336 ÷ 12   × 15 − ? = 138
    (b) Rs. 3750 and Rs. 3000                (a) 140             (b) 233

    (c) Rs. 5000 and Rs. 4000                (c) 420           (d) 282
                                             (d) None of these
    (d) None of these
                                         76. 168   × 15 ÷ 24 × 12 = ?
70. Income of C is 20% more than
    that of B and income of B is 25%         (a) 1160            (b) 8.75
    more than that of A. Find out            (c) 1260            (d) 105
    by how much % is the income
    of C more than that of A.                (e) None of these

    (a) 25%             (b) 75%          77. 4410 ÷ 45 ÷ 7 = ?

    (c) 50%             (d)100%              (a) 98              (b) 686

71. Spending Rs. 1,200 daily for 7           (c) 14              (d) 70
    days, I ran into a debt which            (e) None of these
    was cleared in 9 days after I
    reduced my daily expense to Rs.      78. 7586 + 11254 − ? = 8976
    880. Find my daily income.               (a) 9846            (b) 9764
    (a) Rs. 1,000       (b) Rs. 1,020        (c) 9784            (d) 9864
    (c) Rs. 1,040       (d) Rs. 1,025        (e) None of these
72. The ratio of incomes of A and B      79. 1111 + 12121 + 1020102 = ?
    is 5 : 3 and that their
    expenditures is 8 : 5. If their          (a) 1303334         (b) 1033344
    savings are in the ratio of 2 : 1        (c) 103334          (d) 1033334
    and their total saving is Rs.
    3,600, then the income of A is :         (e) None of these

    (a) Rs. 12,000      (b) Rs. 7,200    80. There are two grades A and B
                                             of workers in a workshop.
    (c) Rs. 7,800       (d) Rs. 9,100        Every worker contributes as
                                             many rupees as there are
73. 1.5   × 1.2 − 0.06 × 0.5 = ?             workers of his own category. If
    (a) 1.77            (b) 17.97            the total amount contributed is
    (c) 1.797           (d) 17.77            Rs. 196 including Rs. 16
                                             contributed by the owner of the
    (e) None of these                        workshop, what is the total
QA – 11 ]                                                                   [ Practice Paper
      number of workers in that                         88. A person wants to divide a sum
      workshop ?                                            of Rs. 3,90,300 between his two
                                                            sons who are 13 and 15 years of
      (a) 18               (b) 14              (c) 12
                                                            age respectively in such a way
      (d) 10               (e) None of these                that their shares, if invested at
                                                            4% per annum compound
        196   17      78
81.         ×      ×      =?                                interest, should produce the
        14    289     169                                   same amount when they
      (a) 1       (b) 2                        (c) 6        become 18 years of age. Find the
      (d) 4               (e) 13                            share of each.
         ?                                                  (a) 187500, 202800
82.            = 250
        .25                                                 (b) 178500, 183000
      (a) 500              (b) 125             (c) 5        (c) 199400, 194500
      (d) 0                (e) 100                          (d) 168390, 195600
      189                                               89. A, B and C invested Rs. 26,000,
83.           = 1.89                                        Rs. 34,000 and Rs. 10,000
        a
                                                            respectively in a business.
      (a) 10               (b) 100         (c) 1000         They earn a profit of Rs. 3500.
      (d) 10000            (e) None of these                B's share of profit is
                                                            (a) Rs. 1200        (b) Rs. 1500
84.     12 + 24 equals
                                                            (c) Rs. 1700        (d) Rs. 1900
      (a) 2 6 + 2 3            (b)   36
                                                        90. Mr. Rai decided to distribute his
      (c)     288              (d) 6 2                      income among the members of
85. Find          100 + 49                                  his family. He gave 50% to his
                                                            wife, 35% of the remaining to
      (a)     149           (b) 17       (c)     490        both of his sons, and the
      (d)                                                   balance of Rs. 6,750 was
              14 + 10 (e) None of these
                                                            deposited by him in the bank.
86.     0.00004761 equals                                   How much amount was received
                                                            by his wife ?
      (a) 0.069                (b) 0.0069
                                                            (a) Rs. 23,200      (b) Rs. 45,000
      (c) 0.00069              (d) 0.0609
                                                            (c) Rs. 22,500      (d) Rs. 13,500
                    3
87. If x =              , find the value of                 (e) None of these
                  2
                                                        91. If a ⊗ b = (a × b) + b, then 5 ⊗ 7
        1 + x+ 1 - x
                                                            equals to
        1 + x− 1 - x
                                                            (a) 12              (b) 35
      (a)     5                (b)   3                      (c) 42              (d) 50
      (c)     2                (d)   4                  92. 24% of 250 + ?% of 240 = 120
Practice Paper (Solved) ]                                                 [ QA – 12
    (a) 25              (b) 40           98. (1502)2 − (1498)2 = ?
    (c) 30              (d) 45                 (a) 12,000          (b) 16,000
    (e) None of these                          (c) 22,56,004       (d) 22,560
93. 22% of ? + 30% of 420 = 192                 3
                                         99.      of 480 ÷ 8 + 82 = ?
    (a) 330             (b) 350                 5
                                               (a) 120            (b) 100
    (c) 200             (d) 280
                                               (c) 36              (d) 44
    (e) None of these
                                         100. Value of 64 ÷ 8 ÷ 4 ÷ 2 is
94. 75   × 18 + ?% of 150 = 1380
                                               (a) 1               (b) 8
    (a) 25              (b) 20
                                               (c) 16              (d) 24
    (c) 12              (d) 16
                                               (e) None of these
    (e) None of these
                                         101. If a : b = 2 : 3, b : c = 5 : 7, then a
95. Subhash bought 20 kg of tea at            : b : c is
    the rate of Rs. 30 per kg and 30
    kg at the rate of Rs. 25 per kg.           (a) 2 : 3 : 7       (b) 2 : 5 : 7
    He mixed the two and sold the              (c) 10 : 15 : 21    (d) 2 : 15 : 7
    mixture at the rate of Rs. 22.50
    per kg. What was his loss in the           (e) None of these
    transaction ?                        102. If two numbers are in the ratio
    (a) Rs. 200         (b) Rs. 225           5 : 7and their least common
                                              multiple is 315, then their
    (c) Rs. 175         (d) Rs. 200.25        product is
96. A boy goes to school with the              (a) 2385            (b) 2538
    speed of 3 km/hr and returns
    with a speed of 2 km/hr. If he             (c) 2358            (d) 2835
    takes 5 hours in all, the distance   103. If x : y = 3 : 4, y : z = 5 : 6 and z : w
    in kms between the village and            = 2 : 3, then x : w equals
    the school is
                                               (a) 5 : 3           (b) 3 : 3
    (a) 6               (b) 7
                                               (c) 5 : 12          (d) 7 : 3
    (c) 8               (d) 9
                                         104. Subtract − 13 from 28 − 5 + 5.
97. Gold is 19 times heavy as water
                                               (a) 51              (b) 53
    and copper 9 times as heavy as
    water. The ratio in which these            (c) 56              (d) 58
    two metals be mixed so that the
                                               (e) None of these
    mixed so that the mixture is 15
    times as heavy as water, is          105. Add 7.007, 70.7 and 7.007
    (a) 1 : 2           (b) 2 : 3              (a) 84.074          (b) 84.714
    (c) 3 : 2           (d) 19 : 135           (c) 84.741          (d) 80.714
QA – 13 ]                                                         [ Practice Paper
106. Which   of    the   following                    2      1
                                         112. Express    of    of Rs. 25.20 as a
     numbers is a multiple of 8 ?                     3      4
    (a) 923862          (b) 923962                         1
                                             fraction of 1 of Rs. 36.
                                                           2
    (c) 923972          (d) 923872
                                                   7                      11
107. If 14% of a number is 105, then         (a)                    (b)
                                                   90                     90
     find the number.
                                                   5                      5
    (a) 715             (b) 705              (c)                    (d)
                                                   8                      42
    (c) 735             (d) 750
                                         113. Simplify :
                                                     LM      R F          I UOP
108. Three pipes fill a tank
     separately in 10 minutes, 20
                                             1+ 2 3− 1+ 2−
                                                             | G
                                                             S H   1 5
                                                                    −     JK |PQ
                                                                             V
     minutes and 30 minutes
     respectively. An outlet pipe can
                                                      MN     |
                                                             T     2 2       |
                                                                             W
     empty it in 15 minutes when no          (a) 2                  (b) 1
     water flows in. If all the pipes        (c) −3                 (d) 5
     are opened, when the tank is
     empty, then how long, in            114. The least of the following
     minutes, will it take to fill the
     tank ?
                                                     2
                                                       b g
                                              0.2, .2 , 0.2 , 1 ÷ 0.2 is

            1                   4            (a) (.2)2              (b) 0.2
    (a) 9               (b) 8
            7                   7            (c) 0.2                (d) 1 ÷ 0.2
           1               2
     (c) 7           (d) 6               115. 5% of 5% of Rs. 100 is
           2               3
109. A number is multiplied by its           (a) Re. 0.25           (b) Re. 0.50
          −
     one−third to get 192. Find the          (c) Rs. 10             (d) Rs. 25
     number.
                                         116. The value of
    (a) 16              (b) 20
    (c) 24              (d) 28                 6 + 6 + 6 + 6+.... is
110. The value of
                                             (a) 2                  (b) 5
         0.125 + 0.027
                             is              (c) 4                  (d) 3
     0.5 × 0.5 + 0.09 − 0.15
                                         117. A cistern is normally filled with
    (a) 1               (b) 0.2
                                              water in 10 hours but takes 5
    (c) 0.08            (d) 0.8               hours longer to fill because of
                                              a leak in its bottom. If the
                    0.01 + 0.0064             cistern is full, then the leak will
111. Evaluate :
                      0.01 × 0.3              empty the cistern in
    (a) 1               (b) 10               (a) 20 hours           (b) 40 hours
    (c) 100             (d) 1000             (c) 50 hours           (d) 30 hours
Practice Paper (Solved) ]                                                                      [ QA – 14
118. Taps A and B can fill a bucket
     in 12 minutes and 15 minutes                                         b12.12g − b8.12g    2         2

     respectively. If both are opened
     and A is closed after 3 minutes,
                                                   120. Simplify
                                                                       b0.25g + b0.25gb19.99g
                                                                                        2


     how much further time would                          LMe8 j OP × 16
                                                                -3/4
                                                                       5/2
                                                                              8/15
                                                                                              3/4

                                                        +N              Q
     it take for B to fill the bucket ?
       (a) 8 min 5 sec.        (b) 8 min 15 sec.
                                                              LMF b128g I OP      3/7
                                                                                            −1/5


                                                               NH         K Q
                                                                             -5
                                                            3
       (c) 7 min 45 sec.       (d) 7 min 15 sec.
119. A and B enter into a
                                                                1                                   1
     partnership investing Rs.                          (a) 4                               (b) 6
     12,000      and      Rs.     16,000                        2                                   2
     respectively. After 8 months, C                        2                 1
     also joins the business with a                     (c) 3           (d) 8
                                                            3                 2
     capital of Rs. 15,000. The share
                                                   121. Find the value of
     of C in a profit of Rs. 45,600 after
                                                                         1                          1
                                                        e28 + 10 3 j − e7 − 4 3 j
     two years is
                                                                         2                          2
       (a) Rs. 12,000          (b) Rs. 14,400
                                                        (a) 5                               (b) 3
       (c) Rs. 19,200          (d) Rs. 21,200
                                                        (c) 8                               (d) 6
       (e) None of these

                        ANSWERS & SOLUTIONS
Ans.1.(a) Let three students are A, B and          Ans.3.(c) There are two ways of
C ⇒ Probability that the given problem             borrowing books
cannot be solved by A, B & C is                    (a) When there is no chemistry book in
    1      2        5   2 3 7        7
FG
 1      IJ FG
        1      1IJ FG     IJ                       this case, 3 books are to be selected from
 H −3
         KH
          −5
                 KH        K
                 − 12 = 3 × 5 × 12 = 30            the remaining 6 books :

⇒ Probability that the problem is solved           This can be done in = 6C3
       7
        FG IJ
      30 = 23
                                                     6 5 4
                                                       × ×
=1−
         H K30
                                                   = 1 2 3 = 20 ways
                                                       × ×
Ans.2.(a) One principal can be                     (b) When there is a chemistry 1 and
appointed out of 36 teachers = 36C1, 36            chemistry 11 books. In this case one
ways.                                              book can be selected from the remaining
For each way of doing so, one vice                 6 books in = 6C1 = 6 ways. Hence, total
                                                   number of ways = 20 + 6 = 26
principal can be appointed out of the
remaining 35 teachers in 35C1 = 35 ways            Ans.4.(d) The equation is same only if
                                                   k=6
Hence, two posts, together can be filled
                                                   (by multiplying the first equation by 2).
in 36 × 35 = 1260 ways.
QA – 15 ]                                                                       [ Practice Paper
Ans.5.(a) Probability of not given                     Ans.13.(e) Given expression
 5 8 7       20 5
   + +                                                     96 3 × 96          6× 6
5 8 7 4 = 24 = 6                                       =                  ×
                                                                3
 + + +                                                   16 × 16       64
Ans.6.(b) 7! = (3C2 × 6! × 2! + 5!)                    simplyfying the expression,
= 5040 − (3 × 720 × 2 + 120) = 1440.                   we get its value as 6.
Ans.7.(b) In the second year the                       Ans.14.(b) If C = 100, A = 72.
investment are :                                       Hence ratio = 25 : 18.
5 × 1.26 : 7 × 1.20 : 6 × 1.15                         Then 18x + 25x + y = 65,000 and 12%
= 6.3 : 8.4 : 6.9 = 21 : 28 : 23.                      (18x) + 18% (25x) + 16% (y) = 10, 180.
                   15
                                                       Solving the two equations,
Ans.8.(a) 36 × 27 × 13 = 260.
                                                       we get y = 22,000.
All other parts equal 240.
                                                       Ans.15.(c) Country the vowels as one,
                           100
Ans.9.(c) MP = 6750 ×             100                  we have 5! ways.
                            90 = 7500.
                                                       Since the vowels can be arranged in 3!
                                × 150 .
Since profit1750
             = 50%, C.P. = 7500                        ways, the reqd. answer is 5! × 3! = 720.
Profit % = 5000 × 100 = 35%.                           Ans.16.(d) Reduce to the base of           2 by
Ans.10.(a) Since there are at least 3                  factorisation and solve.
men in the committee, we can have 3                    Ans.17.(a) Milk left after 4th operation/
cases, either 3 or 4 or 5 men.                         Whole quantity of container
                                                            1 4 16
(i) 7C3 × 6C3                       (ii) 7C4 × 6C1       1 FG       IJ
(iii)   7C
             5   ×   6C
                          0   = 525 + 210 + 21 = 756
                                                       =
                                                            H
                                                           −3 =
                                                                   x K ⇒ x = 81 litres.

Ans.11.(c)
                                                                   p 1 2 FG     IJ  1
                                                                     − p = P 2 + 2 − 2.
I.      9x2 + y2 + 6xy − 5x2 + 2xy
                                                       Ans.18.(a)
                                                                          H      K P

= 4x2 + y2 + 8xy                                       Hence b2 = a − 2.

II.     (2x − y)2 = 4x2 + y2 − 4xy                     Ans.19.(d)

III. 4x2 + y2 + 4xy − 2xy                              a(c + a)(a + b) − b (a +b)(b + c)

= 4x2 + y2 + 2xy                                                                       = abc − abc = 0.

IV. 4x2 + 9y2 + 12xy − 16xy − 8y2                      Ans.20.(b) Suppose the salary was Rs.
                                                       100, to begin with.
3= 4x2 + y2 − 4xy
                                                       ⇒ 100 + 5% = 105
Hence only II and IV are equal.                        ⇒ 105 − 2.5% = 102.375
Ans.12.(c) Salary 2 years back                         If the present salary is Rs. 102.375, then
                     100   100                         the salary in the beginning was Rs. 100.
= 26,640 ×               ×
                     120   120 = 18,500.
Practice Paper (Solved) ]                                                             [ QA – 16
If the present salary Rs. 22702.68, then                    Ans. 25.(c) a + b = 24, a − b = 8
the salary the beginning was
   100                                                      ⇒ a = 16, b = 8    ⇒ ab = 128
 102.375 × 22702.68 = 22176.                                ⇒ Required equation is the one whose
                                                            sum of the roots is 24 and product of the
Ans.21.(b) The sequence in the given
                                                            roots is 128.
series is
                                                            i.e. x2 − 24 x + 128 = 0
× 1 + 12, × 2 + 22, × 3 + 32, × 4 + 42,                                   x
× 5 + 52.                                                                y =4
                                                            Ans.26.(a)        5,x=y−5
Ans.22.(a) The cheque was received for                               4y 4y
Rs. 44.06. After spending Rs. 5.42, he
                                                            ⇒ x= 5 , 5 =y−5
had Rs. 38.64, which is 6 times of Rs.
6.44.                                                       ⇒ y = 25, x = 20 ⇒ x + y = 45
∴ He should have received Rs. 6.44.                         = Total amount
                               c
                       b
Ans.23.(b) α + β = −      αβ = a                            Ans.27.(c) Let the monthly incomes of
                       a,                                   two persons be 5I and 4I and their
α2 + β2 = (α + β)2 − 2αβ                                    monthly expenditures be 9E and 7E
  b       c b       ac                                      respectively.
    2          2
        2        −2
= a − a =        a
      2                    2                                ⇒ 5I − 9E = 500
      α 2 β2 α 3 + β 3                                      ⇒ 4I − 7E = 500
⇒        +   =
      β    α    αβ                                          ⇒ I = 1000, E = 500
    (α + β)(α 2 + β2 − αβ)                                  ⇒ Monthly incomes of the two persons
=                                                           are Rs. 5000 and Rs. 4000.
              αβ
  FG − b IJ FG b   2
                       − 2ac c     I
                                   JK                       Ans.28.(b) 4! × 2 ways,

=
   H aKH               a2
                            −
                              a                             i.e. 24 × 2 = 48 ways.
                       c
                                                            Ans.29.(d) 26 × 12 ÷ 8 + ? = 76
                       a
          2                                 3                       12
 − b (b − 3 a c)               a                abc − b 3   ⇒ 26 ×      + ? = 76
              3            ×            =                            8
          a                    c                 a 2c
                                                            ⇒ ? = 76 − 39 = 37
Ans.24.(d) 3C +2T = 700                                     Ans.30.(a) Change in meter reading
5C + 3T = 1100                                              = 18293 − 17385 = 908 units.
⇒ 9C + 6T = 2100                                            Rounded off to 910 units.
⇒ 10C + 6T = 2200                                           Therefore Bill is (910)(55) = 50050 paise,
⇒ C = 100, T = 200                                          or Rs. 500.50.
⇒ 2C + 2T = 200 + 400 = 600.                                This is rounded off to Rs. 501.
QA – 17 ]                                                         [ Practice Paper
                                                           10
Ans.31.(a) In the long run, every
number will have occurred roughly the     Ans.37.(c) x (5) 2 = 5
same number of times.
                                          Ans.38.(b) Let the no. of crowds on the
Therefore for every 6 throws, each        opening day = x
number from 1 to 6 would have occurred.
                                          Monday = x,               Tuesday = 2x
Therefore the gambler would have won
1 + 2 + 3 + 4 + 5 + 6 i.e. Rs. 21.        Wednesday = 4x,           Thursday = 8x
He would have spent Rs. (6 × 3) = 18.     Friday = 16x,
Therefore gain is Rs. 3 in 6 throws,
                                          Saturday = 32x = 6400
i.e. 50 paise per throw.                         6400
            P
                                          ⇒ x = 32 = 200.
Ans.32.(b) J max ⇒ P max, J min
                                          Ans.39.(c) Let cost of a chair and a table
[P − 10] = 12 ⇒ P = 22 or − 2             be Rs. x and y respectively.
[4J − 10] = 6 ⇒ J = 4 or 1.               Then, 15x + 2y = 4000 .....(i)
                 P
                                          10x = 4y                  ....(ii)
So max. value of J is 22.
                                          Solving (i) and (ii)
Ans.33.(b) x + y > 5                      x = 200, y = 500
x − y > 3, 2x > 8, x > 4.                 12x + 3y = 12 × 200 + 3 × 500
Ans.34.(a) 136     × 25 ÷ 16 × ? = 2550   = Rs. 3900
           25                             Ans.40.(a) 5x + 2y = 1080 .....(i)
⇒ 136 ×    × ? = 2550
        16                                2x = y or 2x − y = 0 ......(ii)
       2550 × 16                          Solving (i) and (ii),
⇒ ? = 136 25 = 12
         ×
                                          x = 120, y = 240.
Ans.35.(c) P(A) = 0.2, P(B) = 0.3
                                          ⇒ 2x + 5y = 2 × 120 + 5 × 240
P(A ∩ B) = P(A).P(B)
                                          = Rs. 1440.
= 0.2 × 0.3 = 0.06
                                          Ans.41.(b) Let his salary = Rs. x
Ans.36.(c)
                                          Expenditure on food
Let the capacity of tank = x liter.             30        3x
3         4          3x 4 x               =x × 100 = Rs. 10
4 x + 5 = 5 x ⇒ 4 − 5 = −5
                                                               3     3x
                                                        x
15x 16x                                   Donation =        × 100 = 100
   −
   20           −x
         = −5 ⇒      −5                     3x    3x
                20 =
                                          ⇒ 10 + 100 = 231
⇒ x = 100
Practice Paper (Solved) ]                                                  [ QA – 18
  30x 3 x                                                    1 4 3 12          3
    100+                                                     4 × 5 × 4 = 80 = 20
⇒         = 231                           Ans.46.(a)
  33 x
                                          Ans.47.(a) Suppose strength of the class
⇒ 100 = 231
                                          = x ⇒ 5x + 10 + 15 = 5.5(x + 2)
⇒ 33x = 231 × 100
            100                           ⇒ 0.5x = 14 ⇒ x = 28
⇒ x = 231 × 33 = 700.                     Ans.48.(b) Given expression
                                            1        (0.67) 3 − (0.1) 3
Ans.42.(b) Let the of ratio of male and   =   +
female is K : 1                             2 (0.67) 2 + 0.67 × 0.1 + (0.1) 2
                                            1
⇒ 5200 K + 4500 = (K + 1) × 5000          = 2 + (0.67 − 0.1)
⇒ 5200 K + 4500 = 5000K + 5000            [ ... a3 − b3 = (a − b)(a2 + b2 + ab)]
⇒ 5200 K − 5000K = 5000 − 4500 = 500      = 0.5 + 0.57 = 1.07
                  500 5
                                          Ans.49.(c) Given expression
200 K = 500 ⇒ K = 200 = 2                   (0.03)     0.01 2 2 0.01 0.03
       5
                                          =
                                                   2
                                                     +      b g
                                                              − ×
                                                           0.02
                                                                    ×
K:1= 2 :1=5:2                                 FG 3 + 3 IJ
= 70 : 30 = 70, 30
                                          ÷
                                               H 8 32 K
Ans.43.(a) Let No. of hens = x              (0.03 − 0.01) 2 32 0.02 × 32
                                                           ×    =
⇒ No. of cows = (48 − x)                  =      0.02        15   15

⇒ 2x + 4 (48 − x) = 140                     0.64    64   16
                                          =      =     =       0.0427
                                             15    1500 375 =
⇒ 2x + 192 − 4x = 140
                                          Ans.50.(d) Given expression
⇒ −2x = 140 − 192 = −52
      52                                       28900000           24 + 24 × 9
                                          =             +
⇒ x = 2 = 26                                    121000                96

Ans.44.(b) Put a = 2, b = 12 in            170 4 24 170          192
                                          =    +       =     +2=     = 17.45
 a                                          11 2 24      11       11
   b = 5b + a2
                                          Ans.51.(c) Required number
     2
⇒        12   = 5 × 12 + 4 = 64           = (4 × 1) + (4 × 2) + (4 × 3) + (1 × 4)
⇒ 26 = 64, which is true.                 = 4 + 8 + 12 + 4 = 28.
Ans.45.(b) No. of bricks required         Ans.52.(b) Total number of alteration
  1500 1200 20                            for 60 shirts
                                              2         3       4
=   36 × 25 70
             ×
                   = 4000
       ×   ×                              = [ 3 × 60 + 4 × 60 + 5 × 60] = 133.
QA – 19 ]                                                                [ Practice Paper
Ans.53.(d) 10 = 10 + 03                       i.e. G + S + C + B = 9.725 ......(i)

⇒ 11 = 10 + 13 ⇒ 18 = 10 + 23                 where G = part of gold in crown
                                              S = part of silver in crown
⇒ 36 = 10 + ?3
                                              C = part of copper in crown
⇒ 74 = 10 + 43
                                              B = part of brass in crown
The number in the series at position 4
must be 10 + 33 = 37.                         Again by the condition of the question :
Therefore the number 36 does not              G + S = 4 kg      .......(ii)
belong to the group.                          G + C = 4.5 kg .......(iii)
Ans.54.(a) Let number of children = x.        G + B = 3.6 kg .......(iv)
So number of mangoes purchased = 2x.
                                              add (ii) + (iii) + (iv)
Number of children present
    x 3x                                      3G + S + C + B = 12.1 kg
=x− 4 = 4 .                                   2G + 9.725 = 12.1 ⇒ 2G = 2.375
                  3x
Number of girls = 4 − 25.                     ⇒ G = 1.1875 kg.
Now by the question,                          Ans.58.(d) Prob. that bag A is drawn =
                                              1

FG 3x − 25IJ × 3 + a25 × 2f = 2x              2 . Prob. that white ball is drawn from
                                                       1
 H4 K                                         bag A = 2 × =
                                                           3
                                                           5 10
                                                                3

⇒ x = 100. So the number of mangoes                                         1
purchased = 200.                              Prob. that bag B is drawn = 2
Ans.55.(c) Suppose the hundred's, ten's       Prob. that white ball is drawn from bag
and unit's places of x be a, b, c                 1 2 1
                                                     × = .
respectively. Make these digits as c, b, a.   B= 2 6 6
Difference of numbers                         Prob. that white ball is drawn either
= (100a + 10b + c) − (100c + 10b + a)         from bag A or from bag B

= 99 (a − c), which is divisible by 9 but       3 1      7
                                              =   + =
none of 4, 6, 12.                              10 6 15
Ans.56.(b) Using a + b + c = 0                Ans.59.(a) P50 (1 − p)50 = P51 (1−P)49
                                                     1
⇒ a3 + b3 + c3 = 3abc,                        ⇒ P=
                                                     2
we get, x1/3 + y1/3 + z1/3 = 0
                                              Ans.60.(b) Total score will be a prime
⇒ x + y + z = 3x1/3 y1/3 z1/3                 number in 15 ways out of 36 :
⇒ (x + y + z)3 = 27 xyz.                      (1,1), (1,2), (1,4), (1,6), (2,1),
Ans.57.(c) Given that the crown made          (2,3), (2,5), (3,2), (3,4), (4,1),
of gold, silver, copper and brass and         (4,3), (5,2), (5,6), (6,1), (6,5).
weighs 9.725 kg,
Practice Paper (Solved) ]                                                                           [ QA – 20
                                                                          4
Hence, the             required                probability
                                                             = 80% of n = 5 n
  15   5
=    =
  36 12                                                      Now, according to the question,
             FG 1 IJ   −2
                                FG 4 IJ   2                                 5    4n
Ans.61.(d)
              H 4K          =
                                 H 1K         = 16           37 + (n − 4) ×
                                                                      5n
                                                                            8 = 5
                                                                          205 4n
Ans.62.(d) There are 12 edges in the
cube, Volume = V.                                            or, 37 + 8 − 8 = 5
                                                                      205  4n 5n
Each edge = V1/3.
                                                             or, 37 − 8 = 5 − 8
Total length of the edges = 12 V1/3.
                                                                 296 205 32    25
Ans.63.(a) 25 × 92 = 32 × 81 = 2592                                 −
                                                                    8        −
                                                                            n40 n
                                                             or,        =
            P x 2 − 36 x + 7    x−6
Ans.64.(a)    = 2      ×      =      .
           Q x − 49 x + 6       x −7                            91 7n
                2    7    5                                        =
           1       FG
               20 + 20 + 20 = 3                 IJ           or 8 91 40
                                                                     40
Ans.65.(b) −        H         10                 K                      ×
                                                             ⇒ n = 8 7 = 65
                                                                        ×
Ans.66.(c) Let the number be N
                                                             Therefore, total number of questions =
⇒ According to the question,                                 65. Thus the required answer is
                                                             option (b).
(44)2 < N < (45)2 ⇒ 1935 < N < 2025
                                                             Ans.68.(a) Complete years 2000. No. of
Therefore, the required number would                         odd−days in 2000 = 0.
be any number between 1937 and 2025.
                                                             Odd day in Jan = 3, Feb = 0, March = 3,
But from the question it is clear that the                   April 6, Total = 12, and odd days = 5.
required number is the factor of 6 and                       Day = Sunday + 5 = Friday
the multiple of 5. So we have to find out
the number between 1937 and 2025                             Ans.69.(c) Let x denote the monthly
which is divisible by both 36 and 5.                         income variable and y denote the
                                                             monthly expenditure.
62 = 36
                                                             As per question,
LCM of 36 and 5 = 36 × 5 = 180
                                                             5        9
180 × 10 = 1800                                                  x−        y = 500 .................(i)
                                                             9        16
180 × 11 = 1980
                                                                   4     7
Thus the required no. is 1980.                               and     x−    y = 500 .........(ii)
                                                                   9    16
So, Answer is (c).                                           Solving (i) and (ii) x = 900
                                                                                        5
Ans.67.(b) Let the total number of
questions asked in examination be n.                         ⇒ Their monthly income are 9 x and
                                                             4
⇒ No. of correct answer                                      9 x =   Rs. 5000 and Rs. 4000
                                                             respectively.
QA – 21 ]                                                              [ Practice Paper
Ans.70.(c) C = B + 20% of B
      B 6B                                   ⇒ 13232 + 1020102 = ?
=B+ 5 = 5 .                                  ⇒ ? = 1033334
                       A    5A
                                             Ans.80.(a) The contribution by the
B = A + 25% of A = A + 4 = 4                 workers in the workshop = 196 − 16 =
        6     6 5       3                    Rs. 180. Let the number of workers in A
⇒ C= 5B= 5×4 A= 2 A                          grade be x and in B grade be y.
      A                                      ∴x2 + y2 = 180.
= A + 2 = A + 50% of A                       Now, by putting x = 12 and y = 6,
Ans.71.(b) Let my daily income be Rs.        we get x2 + y2 = 180
x.
                                             ∴x + y = 12 + 6 = 18
Then, 7 (1200 − x) = 9 (x − 880) x = 1020.
                                                          196   17    78
Ans.72.(a)                                   Ans.81.(c)       ×     ×     =?
                                                          14    289   169
Saving of A = 2400, of B 1200.
                                                      14 × 14    17        78
Then, 5x − 8y = 2400 and 3x − 5y = 1200.     ⇒ ?=             ×         ×
                                                       14       17 × 17   13 × 13
Solving, we get x = 2400.
Hence 5x = 12,000.                                  14    17       78   18564
                                             ⇒ ? = 14 × 17 × 13 = 3094 = 6
Ans.73.(a) ? = 1.5   × 1.2 − 0.06 × 0.5
                                                               ?
= 1.80 − 0.030 = 1.770 = 1.77                Ans.82.(b)             = 250
                                                           0.25
Ans.74.(b)
                                             ⇒ ? = 250 × 0.25
76.59 + 129.052 − 38.314 = ? + 45.72
                                             ⇒ 250 × 0.5 × 0.5 = 250 × 0.5
⇒ 205.642 − 38.314 − 45.72 = ?
                                                       5
⇒ 167.328 − 45.72 = ? ⇒ ? = 121.608          ⇒ 250 ×      = 125
                                                      10
Ans.75.(d) 336 ÷ 12    × 15 − ? = 138        Ans.83.(d)
                                                         189
                                                             = 1.89 ⇒
                                                                      189
                                                                           = a
                                                           a          1.89
⇒ ? = 336/12 × 15 − 138
                                                 18900
= 28 × 15 − 138 = 420 − 138 = 282            ⇒         = a , Squaring both sides
                                                  189
                     15
Ans.76.(c) ? = 168 ×     × 12 = 1260         (100)2 = ( a )2 ⇒ 10000 = a
                     24
Ans.77.(c) 4410 ÷ 45 ÷ 7 = 98 ÷ 7 = 14       Ans.84.(a)   12 + 24
Ans.78.(d) 7586 + 11254 − ? = 8976           = 2×2×3 +         2×2×2×3 = 2 3+2 6
⇒ 18840 − 8976 = ?                           Ans.85.(b)   100 + 49
⇒ ? = 9864                                   ⇒ 10 × 10 + 7 × 7 = 10 + 7= 17
Ans.79.(d) 1111 + 12121 + 1020102 = ?
Practice Paper (Solved) ]                                                                                            [ QA – 22
Ans.86.(b)              0.00004761                                           FG         4     IJ   5
                                                                                                            b           g FGH1 + 100 IJK
                                                                                                                                  4
                                                                                                                                           3


       4761          69 × 69
                                                                              H
                                                                           x 1+
                                                                                       10 0    K       = 390300 − x
=              =
   100000000      10000 × 10000                                            676
                                                                               x = (390300 − x)
    69                                                                     625
=        = 0.0069
  10000
                                                                           ⇒ x = 187500 and y = 390300 − 187500
Ans.87.(b)
                                                                           ⇒ y = 202800
    1 + x + 1− x                    1 + x + 1− x
                              ×                                            Ans.89.(c) Profit of Rs. 3500 is divided
    1 + x − 1−x                     1 + x + 1− x                           among A, B and C in the ratio 26000 :

=
        e    1 + x + 1− x           j   2                                  34000 : 10000.
                                                                           i.e. in the ratio 13 : 17 : 5.
    e       1 + x   j e
                    2
                        −     1− x      j   2

                                                                           ∴B's share of profit =
                                                                                                                17
                                                                                                                   × 3500
                                                                                                                35
    1 + x + 1 − x + 2 1 − x2   1 + 1 − x2
=
            a
        1 + x − 1− xfa f     =
                                   x
                                                                           = Rs. 1700.
                                                                           Ans.90.(c) 15% of the income = Rs. 6750
    1 + 1 − 34               1+ 1                                          ∴ Total income = Rs. 45000
=                       =           2= 3
                3
                3                                                          ⇒ Amount given to the wife by Mr. Rai
        2         2
                                                                           = Rs. 22500
Ans.88.(a) Let x and y be the shares of
the two sons.                                                              Ans.91.(c) 5 ⊗ 7 = (5                   × 7) + 7 = 42
∴x + y = 3,90,300                                                                                  24               ?
                                                                           Ans.92.(a)                    × 250 +
                                                                                                        × 240 = 120
⇒ y = (390300 − x)                                                                      100        100
For the boy of age 13 years,                                                       ? × 12
                                                                           ⇒ 60 +         = 120
= 5 years, Rate = 4%, Principal = x                                                  5
∴Amount after 5 years compounded
                                                                           ⇒? =
                                                                                        b
                                                                                  120 − 60 × 5
                                                                                                = 25
                                                                                                        g
                        FG         4        IJ   5                                      12
annually
                         H
                    = x 1+
                                  100        K                .......(i)
                                                                           Ans.93.(e)
                                                                                         22
                                                                                        100
                                                                                            ×?+
                                                                                                 30
                                                                                                100
                                                                                                     × 420 = 192
For the boy of age 15 years :
                                                                              22 × ?
Time = 3 years, Principal = 390300 − x                                     ⇒         = 192 −126 = 66
                                                                               100
∴Amount after 3 years compounded
annually
                                                                           ⇒ ? = 300
                                                                           Ans.94.(b) ? % of 150 + 75                   × 18 = 1380
    b                   g FGH1 + 100 IJK
                                                 3
                                  4
= 390300 − x                                         ......(ii)                    ?
                                                                           ⇒                × 150 + 1350 = 1380
                                                                                  100
From (i) & (ii), we get                                                                 3000
                                                                           ⇒? =                        ⇒ ? = 20
                                                                                        150
QA – 23 ]                                                          [ Practice Paper
Ans.95.(b) C.P. of 50 kg. of tea           ⇒ 35 K = 315 ⇒ K = 9
= Rs. (20 × 30 + 30 × 25) = Rs. 1350.      Hence the numbers are 45 and 63.
S.P. of 50 kg. of tea = Rs. (50 × 22.50)   Their product is 2835.

= Rs. 1125.                                               X 3 Y 5 Z   2
                                           Ans.103.(c)     = , = ,  =
                                                          Y 4 Z 6 W 3
Loss = Rs. (1350 −1125) = Rs. 225.
                                                X   Y   Z   W
                                           ⇒      =   =   =
           2×3×2   2d                           15 20 24 36
Ans.96.(a)       =
            3+2     5                      ⇒ X : W = 15 : 36 = 5 : 12
LM Average Speed            2xy   OP       Ans.104.(e) 28 − 5 + 5 − (−13) = 41
 N                      =
                            x+y    Q       Ans.105.(b) 7.007 + 70.7 + 7.007 =
∴10 d = 60 or d = 6.                       84.714
Ans.97.(b) Let x gm of water be taken      Ans.106.(d) Last three digits must be
Then, Gold = 19x gm & Copper = 9x gm.      divided by 8.

Let 1 gm of gold be mixed with y gm of     Ans.107.(d) Let x be the number
Copper.                                    14% of x = 105 ⇒ x = 750.
Then, 19x + 9xy = 15x (1 + y)              Ans.108.(b)         In    one     minute,
           FG 2IJ                           1   1   1   1   11 1   7
⇒y =
            H 3K                              +   +   −   =   −
                                           10 20 30 15 60 15 60
                                                                 =

Ans.98.(a) (1502)2 − (1498)2               of the tank can be filled.
= (1502 − 1498) (1502 + 1498)              ∴Whole tank will be filled in
= 4 × 3000 = 12000.
                                           60    4
          3                                   = 8 minutes.
Ans.99.(b) of 480 ÷ 8 + 82 = ?              7    7
          5                                                1
                                                       x     x
       3            1                      Ans.109.(c)   × 3 = 192
∴? =
       5    × 480 × 8 + 64
                                           ⇒ x = 24
= 36 + 64 = 100
                                           Ans.110.(d) Given expression
Ans.100.(a) 64 ÷ 8 ÷ 4 ÷ 2
                                                     0.152
= 8 ÷4 ÷2 = 2 ÷2 = 1                       =
                                               0.25 + 0.09 − 0.15
            a 2 b 5
Ans.101.(c)    = ,    =                        0.152   152   8
             b 3 c 7                       =         =     =   = 0.8
    a   b    c                                  0.19   190 10
⇒     =   =
   10 15 21                                Ans.111.(c) Given expression
Ans.102.(d) Let the two numbers be 5K
and 7K.                                         0.01 + 0.08    0.09    0.3
                                           =                =       =       = 100
∴L C M of 5 K and 7 K = 35 K                      0.003       0.003   0.003
Practice Paper (Solved) ]                                                                    [ QA – 24
Ans.112.(a)                                                 11
                                                ∴Remaining      of the bucket is filled
                                                            20
2    1               3
  of   of 25.20 = K × of 36                                      11
3    4               2                          by tap B in 15 ×    = 8 minutes 15
                                                                 20
⇒ 4.2 = 54 K                                    seconds.
          42   7                                Ans.119.(a) A, B and C's shares in the
⇒ K=         =
         540 90                                 capital are in the ratio of 12000 × 24 :

                   LM R FG             IJ UOP
                                                16000 × 24 : 15000 × 16 ,

                    NM S H              K VQP
                                 1 5
Ans.113.(c) 1 + 2 3 − 1 + 2 −     −             i.e. 288 : 384 : 240,
                       T         2 2      W     i.e. 18 : 24 : 15,
= 1 + 2 [3 − {1 + (2 + 2)}]
                                                i.e. 6 : 8 : 5
= 1 + 2 [3 − 5] = 1 + 2 (−2)
                                                ∴ Share of C in the profit
= 1 − 4 = −3
                                                  5
Ans.114.(a) 0.2, (.2)2 = 0.04,                  =   × 45600 = Rs. 12000
                                                 19
                         1
0.2 = 0.222 , 1 ÷ 0.2 = 0.2 = 5                 Ans.120.(a) Given expression
Ans.115.(a) 5% of (5% of 100)
                                                    b12.12 + 8.12gb12.12 − 8.12g
= 5% of 5 =
               1
                 = 0.25
                                                =
                                                         b0.25gb0.25 + 19.99g
               4
                                                 LM8 OP × FH16 IK
                                                        −15 / 8
                                                                  8 / 15              1/ 4

                                                +N          Q
                                                                                  3
Ans.116.(d) Let x =       6 + 6 + 6+.....
                                                        LMb128g OP   −15/7
                                                                             −1/5
⇒x =           2
         6+x ⇒x = 6 + x                                  N
                                                         3
                                                                    Q
⇒ x2 − x − 6 = 0
                                                         4    8 −1 × 8 = 4 + 1                      =4
                                                                                                         1
⇒ (x − 3)(x + 2) = 0                            =           +                                   1        2
                                                                    b g                  e j
                                                        0.25 3       3
                                                                128 7        27                 7
⇒x = 3
                             1   1   1                                                  1                        1
                               −   =
Ans.117.(d) In one hour,
                                                                      e               j e                j
                                                                                                             −
                            10 15 30            Ans.121.(b) 28 + 10 3                   2    − 7−4 3             2

of the cistern will be empty.
                                                  Re5 + 3 j U − Re2 −                    U
                                                                    1                      -1

Ans.118.(b) Tap A and Tap B can fill            = S
                                                  T          V S
                                                             W T
                                                                  2 2
                                                                                      3j V
                                                                                         2 2

                                                                                         W
          1   1   2
                                                = e5 + 3 j − e2 − 3 j
            +   =                                                                −1
together            of the bucket in
         12 15 30
one minute.
                                                = e5 + 3 j −
                                                                1     2+                 3
                                                                    ×
                     9                                       2− 3 2+                     3
In three minutes,       of the bucket is
                    20
filled by taps A and B.
                                                    e         j e
                                                = 5+ 3 − 2+ 3 = 3            j

More Related Content

What's hot

Online examination system Documentation
Online examination system DocumentationOnline examination system Documentation
Online examination system Documentation
LehlohonoloMakoti
 
ISTQB Technical Test Analyst 2012 Training - The Technical Test Analyst's Tas...
ISTQB Technical Test Analyst 2012 Training - The Technical Test Analyst's Tas...ISTQB Technical Test Analyst 2012 Training - The Technical Test Analyst's Tas...
ISTQB Technical Test Analyst 2012 Training - The Technical Test Analyst's Tas...
Amr Ali (ISTQB CTAL Full, CSM, ITIL Foundation)
 
Software testing objective_types
Software testing objective_typesSoftware testing objective_types
Software testing objective_types
sangeeswaran
 
40 ĐỀ LUYỆN THI ĐÁNH GIÁ NĂNG LỰC ĐẠI HỌC QUỐC GIA THÀNH PHỐ HỒ CHÍ MINH NĂM ...
40 ĐỀ LUYỆN THI ĐÁNH GIÁ NĂNG LỰC ĐẠI HỌC QUỐC GIA THÀNH PHỐ HỒ CHÍ MINH NĂM ...40 ĐỀ LUYỆN THI ĐÁNH GIÁ NĂNG LỰC ĐẠI HỌC QUỐC GIA THÀNH PHỐ HỒ CHÍ MINH NĂM ...
40 ĐỀ LUYỆN THI ĐÁNH GIÁ NĂNG LỰC ĐẠI HỌC QUỐC GIA THÀNH PHỐ HỒ CHÍ MINH NĂM ...
Nguyen Thanh Tu Collection
 
Software testing vs. Game testing
Software testing vs. Game testingSoftware testing vs. Game testing
Software testing vs. Game testing
Johan Hoberg
 
The tich khoi chop va khoi lang tru le minh tien
The tich khoi chop va khoi lang tru le minh tienThe tich khoi chop va khoi lang tru le minh tien
The tich khoi chop va khoi lang tru le minh tien
quanvfu
 
Time Table Management System
Time Table Management SystemTime Table Management System
Time Table Management System
Muhammad Zeeshan
 
TỔNG HỢP ĐỀ KIỂM TRA 1 TIẾT CHƯƠNG 1 TOÁN 6 CÓ LỜI GIẢI 2015 - 2016
TỔNG HỢP ĐỀ KIỂM TRA 1 TIẾT CHƯƠNG 1 TOÁN 6 CÓ LỜI GIẢI 2015 - 2016TỔNG HỢP ĐỀ KIỂM TRA 1 TIẾT CHƯƠNG 1 TOÁN 6 CÓ LỜI GIẢI 2015 - 2016
TỔNG HỢP ĐỀ KIỂM TRA 1 TIẾT CHƯƠNG 1 TOÁN 6 CÓ LỜI GIẢI 2015 - 2016
Hoàng Thái Việt
 
Online quiz system
Online quiz systemOnline quiz system
Online quiz system
roshidayu ibrahim
 
Test Estimation Techniques
Test Estimation TechniquesTest Estimation Techniques
Test Estimation TechniquesNishant Worah
 
Online Examination System Report
Online Examination System ReportOnline Examination System Report
Online Examination System ReportAnkan Banerjee
 
Đề Thi HK2 Toán 6 - THCS Đống Đa
Đề Thi HK2 Toán 6 - THCS Đống ĐaĐề Thi HK2 Toán 6 - THCS Đống Đa
Đề Thi HK2 Toán 6 - THCS Đống Đa
Trung Tâm Gia Sư Việt Trí
 
Online course registration system development software engineering project pr...
Online course registration system development software engineering project pr...Online course registration system development software engineering project pr...
Online course registration system development software engineering project pr...
MD.HABIBUR Rahman
 
Đề Thi HK2 Toán 6 - THCS Đoàn Thị Điểm
Đề Thi HK2 Toán 6 - THCS Đoàn Thị ĐiểmĐề Thi HK2 Toán 6 - THCS Đoàn Thị Điểm
Đề Thi HK2 Toán 6 - THCS Đoàn Thị Điểm
Trung Tâm Gia Sư Việt Trí
 
Online Examination System
Online Examination SystemOnline Examination System
Online Examination System
Samrat Roy
 
Program Check List (PCL) FSOFT
Program Check List (PCL) FSOFTProgram Check List (PCL) FSOFT
Program Check List (PCL) FSOFT
Mobi Marketing
 
ĐỀ THI SEAMO - CẤP ĐỘ C (KHỐI 5 + 6) NĂM 2018
ĐỀ THI SEAMO - CẤP ĐỘ C (KHỐI 5 + 6) NĂM 2018ĐỀ THI SEAMO - CẤP ĐỘ C (KHỐI 5 + 6) NĂM 2018
ĐỀ THI SEAMO - CẤP ĐỘ C (KHỐI 5 + 6) NĂM 2018
Bồi Dưỡng HSG Toán Lớp 3
 
Student Database Management System
Student Database Management SystemStudent Database Management System
Student Database Management System
Ajay Bidyarthy
 
Database Assignment
Database AssignmentDatabase Assignment
Database Assignment
Jayed Imran
 
ĐỀ THI HK2 LỚP 1 TOÁN - TV THEO TT22 THAM KHẢO
ĐỀ THI HK2 LỚP 1 TOÁN - TV THEO TT22 THAM KHẢOĐỀ THI HK2 LỚP 1 TOÁN - TV THEO TT22 THAM KHẢO
ĐỀ THI HK2 LỚP 1 TOÁN - TV THEO TT22 THAM KHẢO
Bồi Dưỡng HSG Toán Lớp 3
 

What's hot (20)

Online examination system Documentation
Online examination system DocumentationOnline examination system Documentation
Online examination system Documentation
 
ISTQB Technical Test Analyst 2012 Training - The Technical Test Analyst's Tas...
ISTQB Technical Test Analyst 2012 Training - The Technical Test Analyst's Tas...ISTQB Technical Test Analyst 2012 Training - The Technical Test Analyst's Tas...
ISTQB Technical Test Analyst 2012 Training - The Technical Test Analyst's Tas...
 
Software testing objective_types
Software testing objective_typesSoftware testing objective_types
Software testing objective_types
 
40 ĐỀ LUYỆN THI ĐÁNH GIÁ NĂNG LỰC ĐẠI HỌC QUỐC GIA THÀNH PHỐ HỒ CHÍ MINH NĂM ...
40 ĐỀ LUYỆN THI ĐÁNH GIÁ NĂNG LỰC ĐẠI HỌC QUỐC GIA THÀNH PHỐ HỒ CHÍ MINH NĂM ...40 ĐỀ LUYỆN THI ĐÁNH GIÁ NĂNG LỰC ĐẠI HỌC QUỐC GIA THÀNH PHỐ HỒ CHÍ MINH NĂM ...
40 ĐỀ LUYỆN THI ĐÁNH GIÁ NĂNG LỰC ĐẠI HỌC QUỐC GIA THÀNH PHỐ HỒ CHÍ MINH NĂM ...
 
Software testing vs. Game testing
Software testing vs. Game testingSoftware testing vs. Game testing
Software testing vs. Game testing
 
The tich khoi chop va khoi lang tru le minh tien
The tich khoi chop va khoi lang tru le minh tienThe tich khoi chop va khoi lang tru le minh tien
The tich khoi chop va khoi lang tru le minh tien
 
Time Table Management System
Time Table Management SystemTime Table Management System
Time Table Management System
 
TỔNG HỢP ĐỀ KIỂM TRA 1 TIẾT CHƯƠNG 1 TOÁN 6 CÓ LỜI GIẢI 2015 - 2016
TỔNG HỢP ĐỀ KIỂM TRA 1 TIẾT CHƯƠNG 1 TOÁN 6 CÓ LỜI GIẢI 2015 - 2016TỔNG HỢP ĐỀ KIỂM TRA 1 TIẾT CHƯƠNG 1 TOÁN 6 CÓ LỜI GIẢI 2015 - 2016
TỔNG HỢP ĐỀ KIỂM TRA 1 TIẾT CHƯƠNG 1 TOÁN 6 CÓ LỜI GIẢI 2015 - 2016
 
Online quiz system
Online quiz systemOnline quiz system
Online quiz system
 
Test Estimation Techniques
Test Estimation TechniquesTest Estimation Techniques
Test Estimation Techniques
 
Online Examination System Report
Online Examination System ReportOnline Examination System Report
Online Examination System Report
 
Đề Thi HK2 Toán 6 - THCS Đống Đa
Đề Thi HK2 Toán 6 - THCS Đống ĐaĐề Thi HK2 Toán 6 - THCS Đống Đa
Đề Thi HK2 Toán 6 - THCS Đống Đa
 
Online course registration system development software engineering project pr...
Online course registration system development software engineering project pr...Online course registration system development software engineering project pr...
Online course registration system development software engineering project pr...
 
Đề Thi HK2 Toán 6 - THCS Đoàn Thị Điểm
Đề Thi HK2 Toán 6 - THCS Đoàn Thị ĐiểmĐề Thi HK2 Toán 6 - THCS Đoàn Thị Điểm
Đề Thi HK2 Toán 6 - THCS Đoàn Thị Điểm
 
Online Examination System
Online Examination SystemOnline Examination System
Online Examination System
 
Program Check List (PCL) FSOFT
Program Check List (PCL) FSOFTProgram Check List (PCL) FSOFT
Program Check List (PCL) FSOFT
 
ĐỀ THI SEAMO - CẤP ĐỘ C (KHỐI 5 + 6) NĂM 2018
ĐỀ THI SEAMO - CẤP ĐỘ C (KHỐI 5 + 6) NĂM 2018ĐỀ THI SEAMO - CẤP ĐỘ C (KHỐI 5 + 6) NĂM 2018
ĐỀ THI SEAMO - CẤP ĐỘ C (KHỐI 5 + 6) NĂM 2018
 
Student Database Management System
Student Database Management SystemStudent Database Management System
Student Database Management System
 
Database Assignment
Database AssignmentDatabase Assignment
Database Assignment
 
ĐỀ THI HK2 LỚP 1 TOÁN - TV THEO TT22 THAM KHẢO
ĐỀ THI HK2 LỚP 1 TOÁN - TV THEO TT22 THAM KHẢOĐỀ THI HK2 LỚP 1 TOÁN - TV THEO TT22 THAM KHẢO
ĐỀ THI HK2 LỚP 1 TOÁN - TV THEO TT22 THAM KHẢO
 

Viewers also liked

Compound interest and related problems in business mathematics
Compound interest and related problems in business mathematics Compound interest and related problems in business mathematics
Compound interest and related problems in business mathematics Dr. Trilok Kumar Jain
 
Maths shortcut
Maths shortcutMaths shortcut
Maths shortcut
Pawan Mishra
 
Percentages average r&p
Percentages average r&pPercentages average r&p
Percentages average r&p
Sreeram Madhav
 
450 synonym antonym questions
450 synonym antonym questions450 synonym antonym questions
450 synonym antonym questions
StudyIQ
 
Numerical reasoning I
Numerical reasoning INumerical reasoning I
Numerical reasoning I
Prasanth Kumar RAGUPATHY
 

Viewers also liked (8)

Compound interest and related problems in business mathematics
Compound interest and related problems in business mathematics Compound interest and related problems in business mathematics
Compound interest and related problems in business mathematics
 
Management Aptitude Test 6 December
Management Aptitude Test 6 DecemberManagement Aptitude Test 6 December
Management Aptitude Test 6 December
 
11. profit & loss
11. profit & loss11. profit & loss
11. profit & loss
 
Maths shortcut
Maths shortcutMaths shortcut
Maths shortcut
 
Percentages average r&p
Percentages average r&pPercentages average r&p
Percentages average r&p
 
450 synonym antonym questions
450 synonym antonym questions450 synonym antonym questions
450 synonym antonym questions
 
Partnership accounts
Partnership accountsPartnership accounts
Partnership accounts
 
Numerical reasoning I
Numerical reasoning INumerical reasoning I
Numerical reasoning I
 

Similar to Chandresh agrawal 121 questions

Bank
BankBank
Nift sample paper
Nift sample paperNift sample paper
Nift sample paper
Academy of Fashion & Design
 
NIFT Test series: Sample GAT paper of fashion design1
NIFT Test series: Sample GAT paper of fashion design1NIFT Test series: Sample GAT paper of fashion design1
NIFT Test series: Sample GAT paper of fashion design1
Academy of Fashion & Design
 
Hexaware mock test1
Hexaware mock test1Hexaware mock test1
Hexaware mock test1
Pantech ProEd Pvt Ltd
 
Previous Years Solved Question Papers for Staff Selection Commission (SSC)…
Previous Years Solved Question Papers for Staff Selection Commission (SSC)…Previous Years Solved Question Papers for Staff Selection Commission (SSC)…
Previous Years Solved Question Papers for Staff Selection Commission (SSC)…
SmartPrep Education
 
Ppnet Numerical Aptitude Rrb Ii
Ppnet Numerical Aptitude Rrb IiPpnet Numerical Aptitude Rrb Ii
Ppnet Numerical Aptitude Rrb Iiguestff824c44
 
Cpt 2009 qa
Cpt 2009 qaCpt 2009 qa
Cpt 2009 qa
ashit smile
 
Linear equations
Linear equationsLinear equations
Linear equations
sangemrameshkumar
 
Class 10 Maths
Class 10 MathsClass 10 Maths
Class 10 Maths
Entrancei
 
imo worksheet for class 9 and 10
imo worksheet for class 9 and 10imo worksheet for class 9 and 10
imo worksheet for class 9 and 10
nitishguptamaps
 
6th grade math review worksheet(1)
6th grade math review worksheet(1)  6th grade math review worksheet(1)
6th grade math review worksheet(1)
dihelp2learn
 
Final examination 2011 class vii
Final examination 2011 class viiFinal examination 2011 class vii
Final examination 2011 class vii
Asad Shafat
 
Sslc maths-5-model-question-papers-english-medium
Sslc maths-5-model-question-papers-english-mediumSslc maths-5-model-question-papers-english-medium
Sslc maths-5-model-question-papers-english-mediummohanavaradhan777
 
TCS - sample questions
TCS - sample questionsTCS - sample questions
TCS - sample questions
Taral Soliya
 
Final examination 2011 class viii
Final examination 2011 class viiiFinal examination 2011 class viii
Final examination 2011 class viii
Asad Shafat
 
E C M2221 P R O B A B I L I T Y A N D S T A T I S T I C S Set1
E C M2221  P R O B A B I L I T Y  A N D  S T A T I S T I C S Set1E C M2221  P R O B A B I L I T Y  A N D  S T A T I S T I C S Set1
E C M2221 P R O B A B I L I T Y A N D S T A T I S T I C S Set1
guestd436758
 
2011 10 lyp_mathematics_sa1_15 (1)
2011 10 lyp_mathematics_sa1_15 (1)2011 10 lyp_mathematics_sa1_15 (1)
2011 10 lyp_mathematics_sa1_15 (1)Tarun Gehlot
 

Similar to Chandresh agrawal 121 questions (20)

Cat Qp
Cat QpCat Qp
Cat Qp
 
Bank
BankBank
Bank
 
Nift sample paper
Nift sample paperNift sample paper
Nift sample paper
 
NIFT Test series: Sample GAT paper of fashion design1
NIFT Test series: Sample GAT paper of fashion design1NIFT Test series: Sample GAT paper of fashion design1
NIFT Test series: Sample GAT paper of fashion design1
 
Hexaware mock test1
Hexaware mock test1Hexaware mock test1
Hexaware mock test1
 
Previous Years Solved Question Papers for Staff Selection Commission (SSC)…
Previous Years Solved Question Papers for Staff Selection Commission (SSC)…Previous Years Solved Question Papers for Staff Selection Commission (SSC)…
Previous Years Solved Question Papers for Staff Selection Commission (SSC)…
 
Ppnet Numerical Aptitude Rrb Ii
Ppnet Numerical Aptitude Rrb IiPpnet Numerical Aptitude Rrb Ii
Ppnet Numerical Aptitude Rrb Ii
 
Cpt 2009 qa
Cpt 2009 qaCpt 2009 qa
Cpt 2009 qa
 
Linear equations
Linear equationsLinear equations
Linear equations
 
Class 10 Maths
Class 10 MathsClass 10 Maths
Class 10 Maths
 
imo worksheet for class 9 and 10
imo worksheet for class 9 and 10imo worksheet for class 9 and 10
imo worksheet for class 9 and 10
 
cxc.Mathsexam1
cxc.Mathsexam1cxc.Mathsexam1
cxc.Mathsexam1
 
Nift ug design gat question paper 2
Nift ug design gat question paper 2Nift ug design gat question paper 2
Nift ug design gat question paper 2
 
6th grade math review worksheet(1)
6th grade math review worksheet(1)  6th grade math review worksheet(1)
6th grade math review worksheet(1)
 
Final examination 2011 class vii
Final examination 2011 class viiFinal examination 2011 class vii
Final examination 2011 class vii
 
Sslc maths-5-model-question-papers-english-medium
Sslc maths-5-model-question-papers-english-mediumSslc maths-5-model-question-papers-english-medium
Sslc maths-5-model-question-papers-english-medium
 
TCS - sample questions
TCS - sample questionsTCS - sample questions
TCS - sample questions
 
Final examination 2011 class viii
Final examination 2011 class viiiFinal examination 2011 class viii
Final examination 2011 class viii
 
E C M2221 P R O B A B I L I T Y A N D S T A T I S T I C S Set1
E C M2221  P R O B A B I L I T Y  A N D  S T A T I S T I C S Set1E C M2221  P R O B A B I L I T Y  A N D  S T A T I S T I C S Set1
E C M2221 P R O B A B I L I T Y A N D S T A T I S T I C S Set1
 
2011 10 lyp_mathematics_sa1_15 (1)
2011 10 lyp_mathematics_sa1_15 (1)2011 10 lyp_mathematics_sa1_15 (1)
2011 10 lyp_mathematics_sa1_15 (1)
 

Recently uploaded

How to Break the cycle of negative Thoughts
How to Break the cycle of negative ThoughtsHow to Break the cycle of negative Thoughts
How to Break the cycle of negative Thoughts
Col Mukteshwar Prasad
 
special B.ed 2nd year old paper_20240531.pdf
special B.ed 2nd year old paper_20240531.pdfspecial B.ed 2nd year old paper_20240531.pdf
special B.ed 2nd year old paper_20240531.pdf
Special education needs
 
Home assignment II on Spectroscopy 2024 Answers.pdf
Home assignment II on Spectroscopy 2024 Answers.pdfHome assignment II on Spectroscopy 2024 Answers.pdf
Home assignment II on Spectroscopy 2024 Answers.pdf
Tamralipta Mahavidyalaya
 
Synthetic Fiber Construction in lab .pptx
Synthetic Fiber Construction in lab .pptxSynthetic Fiber Construction in lab .pptx
Synthetic Fiber Construction in lab .pptx
Pavel ( NSTU)
 
Students, digital devices and success - Andreas Schleicher - 27 May 2024..pptx
Students, digital devices and success - Andreas Schleicher - 27 May 2024..pptxStudents, digital devices and success - Andreas Schleicher - 27 May 2024..pptx
Students, digital devices and success - Andreas Schleicher - 27 May 2024..pptx
EduSkills OECD
 
Operation Blue Star - Saka Neela Tara
Operation Blue Star   -  Saka Neela TaraOperation Blue Star   -  Saka Neela Tara
Operation Blue Star - Saka Neela Tara
Balvir Singh
 
CLASS 11 CBSE B.St Project AIDS TO TRADE - INSURANCE
CLASS 11 CBSE B.St Project AIDS TO TRADE - INSURANCECLASS 11 CBSE B.St Project AIDS TO TRADE - INSURANCE
CLASS 11 CBSE B.St Project AIDS TO TRADE - INSURANCE
BhavyaRajput3
 
Template Jadual Bertugas Kelas (Boleh Edit)
Template Jadual Bertugas Kelas (Boleh Edit)Template Jadual Bertugas Kelas (Boleh Edit)
Template Jadual Bertugas Kelas (Boleh Edit)
rosedainty
 
Palestine last event orientationfvgnh .pptx
Palestine last event orientationfvgnh .pptxPalestine last event orientationfvgnh .pptx
Palestine last event orientationfvgnh .pptx
RaedMohamed3
 
TESDA TM1 REVIEWER FOR NATIONAL ASSESSMENT WRITTEN AND ORAL QUESTIONS WITH A...
TESDA TM1 REVIEWER  FOR NATIONAL ASSESSMENT WRITTEN AND ORAL QUESTIONS WITH A...TESDA TM1 REVIEWER  FOR NATIONAL ASSESSMENT WRITTEN AND ORAL QUESTIONS WITH A...
TESDA TM1 REVIEWER FOR NATIONAL ASSESSMENT WRITTEN AND ORAL QUESTIONS WITH A...
EugeneSaldivar
 
aaaaaaaaaaaaaaaaaaaaaaaaaaaaaaaaaaaaaaaaaaaaaaaaaaaaaaa
aaaaaaaaaaaaaaaaaaaaaaaaaaaaaaaaaaaaaaaaaaaaaaaaaaaaaaaaaaaaaaaaaaaaaaaaaaaaaaaaaaaaaaaaaaaaaaaaaaaaaaaaaaaaaa
aaaaaaaaaaaaaaaaaaaaaaaaaaaaaaaaaaaaaaaaaaaaaaaaaaaaaaa
siemaillard
 
Mule 4.6 & Java 17 Upgrade | MuleSoft Mysore Meetup #46
Mule 4.6 & Java 17 Upgrade | MuleSoft Mysore Meetup #46Mule 4.6 & Java 17 Upgrade | MuleSoft Mysore Meetup #46
Mule 4.6 & Java 17 Upgrade | MuleSoft Mysore Meetup #46
MysoreMuleSoftMeetup
 
The Art Pastor's Guide to Sabbath | Steve Thomason
The Art Pastor's Guide to Sabbath | Steve ThomasonThe Art Pastor's Guide to Sabbath | Steve Thomason
The Art Pastor's Guide to Sabbath | Steve Thomason
Steve Thomason
 
Language Across the Curriculm LAC B.Ed.
Language Across the  Curriculm LAC B.Ed.Language Across the  Curriculm LAC B.Ed.
Language Across the Curriculm LAC B.Ed.
Atul Kumar Singh
 
Unit 2- Research Aptitude (UGC NET Paper I).pdf
Unit 2- Research Aptitude (UGC NET Paper I).pdfUnit 2- Research Aptitude (UGC NET Paper I).pdf
Unit 2- Research Aptitude (UGC NET Paper I).pdf
Thiyagu K
 
Phrasal Verbs.XXXXXXXXXXXXXXXXXXXXXXXXXX
Phrasal Verbs.XXXXXXXXXXXXXXXXXXXXXXXXXXPhrasal Verbs.XXXXXXXXXXXXXXXXXXXXXXXXXX
Phrasal Verbs.XXXXXXXXXXXXXXXXXXXXXXXXXX
MIRIAMSALINAS13
 
Unit 8 - Information and Communication Technology (Paper I).pdf
Unit 8 - Information and Communication Technology (Paper I).pdfUnit 8 - Information and Communication Technology (Paper I).pdf
Unit 8 - Information and Communication Technology (Paper I).pdf
Thiyagu K
 
The Challenger.pdf DNHS Official Publication
The Challenger.pdf DNHS Official PublicationThe Challenger.pdf DNHS Official Publication
The Challenger.pdf DNHS Official Publication
Delapenabediema
 
Additional Benefits for Employee Website.pdf
Additional Benefits for Employee Website.pdfAdditional Benefits for Employee Website.pdf
Additional Benefits for Employee Website.pdf
joachimlavalley1
 
Welcome to TechSoup New Member Orientation and Q&A (May 2024).pdf
Welcome to TechSoup   New Member Orientation and Q&A (May 2024).pdfWelcome to TechSoup   New Member Orientation and Q&A (May 2024).pdf
Welcome to TechSoup New Member Orientation and Q&A (May 2024).pdf
TechSoup
 

Recently uploaded (20)

How to Break the cycle of negative Thoughts
How to Break the cycle of negative ThoughtsHow to Break the cycle of negative Thoughts
How to Break the cycle of negative Thoughts
 
special B.ed 2nd year old paper_20240531.pdf
special B.ed 2nd year old paper_20240531.pdfspecial B.ed 2nd year old paper_20240531.pdf
special B.ed 2nd year old paper_20240531.pdf
 
Home assignment II on Spectroscopy 2024 Answers.pdf
Home assignment II on Spectroscopy 2024 Answers.pdfHome assignment II on Spectroscopy 2024 Answers.pdf
Home assignment II on Spectroscopy 2024 Answers.pdf
 
Synthetic Fiber Construction in lab .pptx
Synthetic Fiber Construction in lab .pptxSynthetic Fiber Construction in lab .pptx
Synthetic Fiber Construction in lab .pptx
 
Students, digital devices and success - Andreas Schleicher - 27 May 2024..pptx
Students, digital devices and success - Andreas Schleicher - 27 May 2024..pptxStudents, digital devices and success - Andreas Schleicher - 27 May 2024..pptx
Students, digital devices and success - Andreas Schleicher - 27 May 2024..pptx
 
Operation Blue Star - Saka Neela Tara
Operation Blue Star   -  Saka Neela TaraOperation Blue Star   -  Saka Neela Tara
Operation Blue Star - Saka Neela Tara
 
CLASS 11 CBSE B.St Project AIDS TO TRADE - INSURANCE
CLASS 11 CBSE B.St Project AIDS TO TRADE - INSURANCECLASS 11 CBSE B.St Project AIDS TO TRADE - INSURANCE
CLASS 11 CBSE B.St Project AIDS TO TRADE - INSURANCE
 
Template Jadual Bertugas Kelas (Boleh Edit)
Template Jadual Bertugas Kelas (Boleh Edit)Template Jadual Bertugas Kelas (Boleh Edit)
Template Jadual Bertugas Kelas (Boleh Edit)
 
Palestine last event orientationfvgnh .pptx
Palestine last event orientationfvgnh .pptxPalestine last event orientationfvgnh .pptx
Palestine last event orientationfvgnh .pptx
 
TESDA TM1 REVIEWER FOR NATIONAL ASSESSMENT WRITTEN AND ORAL QUESTIONS WITH A...
TESDA TM1 REVIEWER  FOR NATIONAL ASSESSMENT WRITTEN AND ORAL QUESTIONS WITH A...TESDA TM1 REVIEWER  FOR NATIONAL ASSESSMENT WRITTEN AND ORAL QUESTIONS WITH A...
TESDA TM1 REVIEWER FOR NATIONAL ASSESSMENT WRITTEN AND ORAL QUESTIONS WITH A...
 
aaaaaaaaaaaaaaaaaaaaaaaaaaaaaaaaaaaaaaaaaaaaaaaaaaaaaaa
aaaaaaaaaaaaaaaaaaaaaaaaaaaaaaaaaaaaaaaaaaaaaaaaaaaaaaaaaaaaaaaaaaaaaaaaaaaaaaaaaaaaaaaaaaaaaaaaaaaaaaaaaaaaaa
aaaaaaaaaaaaaaaaaaaaaaaaaaaaaaaaaaaaaaaaaaaaaaaaaaaaaaa
 
Mule 4.6 & Java 17 Upgrade | MuleSoft Mysore Meetup #46
Mule 4.6 & Java 17 Upgrade | MuleSoft Mysore Meetup #46Mule 4.6 & Java 17 Upgrade | MuleSoft Mysore Meetup #46
Mule 4.6 & Java 17 Upgrade | MuleSoft Mysore Meetup #46
 
The Art Pastor's Guide to Sabbath | Steve Thomason
The Art Pastor's Guide to Sabbath | Steve ThomasonThe Art Pastor's Guide to Sabbath | Steve Thomason
The Art Pastor's Guide to Sabbath | Steve Thomason
 
Language Across the Curriculm LAC B.Ed.
Language Across the  Curriculm LAC B.Ed.Language Across the  Curriculm LAC B.Ed.
Language Across the Curriculm LAC B.Ed.
 
Unit 2- Research Aptitude (UGC NET Paper I).pdf
Unit 2- Research Aptitude (UGC NET Paper I).pdfUnit 2- Research Aptitude (UGC NET Paper I).pdf
Unit 2- Research Aptitude (UGC NET Paper I).pdf
 
Phrasal Verbs.XXXXXXXXXXXXXXXXXXXXXXXXXX
Phrasal Verbs.XXXXXXXXXXXXXXXXXXXXXXXXXXPhrasal Verbs.XXXXXXXXXXXXXXXXXXXXXXXXXX
Phrasal Verbs.XXXXXXXXXXXXXXXXXXXXXXXXXX
 
Unit 8 - Information and Communication Technology (Paper I).pdf
Unit 8 - Information and Communication Technology (Paper I).pdfUnit 8 - Information and Communication Technology (Paper I).pdf
Unit 8 - Information and Communication Technology (Paper I).pdf
 
The Challenger.pdf DNHS Official Publication
The Challenger.pdf DNHS Official PublicationThe Challenger.pdf DNHS Official Publication
The Challenger.pdf DNHS Official Publication
 
Additional Benefits for Employee Website.pdf
Additional Benefits for Employee Website.pdfAdditional Benefits for Employee Website.pdf
Additional Benefits for Employee Website.pdf
 
Welcome to TechSoup New Member Orientation and Q&A (May 2024).pdf
Welcome to TechSoup   New Member Orientation and Q&A (May 2024).pdfWelcome to TechSoup   New Member Orientation and Q&A (May 2024).pdf
Welcome to TechSoup New Member Orientation and Q&A (May 2024).pdf
 

Chandresh agrawal 121 questions

  • 1. QA – 1 ] || Ohm Sai Ram || [ Practice Paper FM Chandresh Agrawal's bestsellers Practice Paper Quantitative Aptitude & Numerical Ability www.clhandreshagrawalbooks.com
  • 2. Practice Paper (Solved) ] [ QA – 2 Practice Paper (Solved) QUANTITATIVE APTITUDE 3 3 1. Three students try to solve a problem independently with a (a) k = 2 (b) k ≠ 2 probability of solving it as (c) k ≠ 6 (d) k = 6 1 2 5 , , respectively. What is 5. A bag contains 5 red balls and 8 3 5 12 blue balls. It also contains 4 the probability that the green and 7 black balls. If a ball problem is solved ? is drawn at randomly find the 23 probability that it is not green 7 (a) 30 (b) 5 1 30 6 32 30 (a) (b) 4 (c) 30 (d) 7 1 7 (c) 6 (d) 4 2. From among 36 teachers in a school one principal and one vice principal are to be 6. The letters of the word appointed. In how many ways PROMISE are arranged so that can this be done ? no two of the vowels should come together. Find total (a) 1260 (b) 1250 number of arrangements. (c) 1240 (d) 1800 (a) 49 (b) 1440 3. A boy has 3 library tickets and (c) 7 (d) 1898 8 books of his interest in the library. Of these 8, he does not 7. Surendra, Rajendra and want to borrow chemistry part Manindra invested some II, unless chemistry part I is amount in a business in the also borrowed. In how many ratio of 5 :7 : 6 respectively. In ways can he choose the three the next year they increased books to be borrowed ? their investments by 26%, 20% and 15% respectively. The profit (a) 56 (b) 27 earned during the second year (c) 26 (d) 41 should be distributed in what ratio among Surendra, 4. If the system 2x + 3y − 5 = 0, 4x + Rajendra and Manindra ky − 10 = 0 has an infinite respectively ? number of solutions, then (a) 31 : 27 : 21 (b) 21 : 28 : 23
  • 3. QA – 3 ] [ Practice Paper (c) 26 : 20 : 15 (c) 564 (d) 645 (d) Cannot be determined (e) None of these (e) None of these 11. Which of the following expression are exactly equal in 8. Four of the following five parts value ? numbered (i), (ii), (iii), (iv) and (v) are exactly equal. The I. (3x − y)2 − (5x2 − 2xy) number of the part which is not II. (2x − y)2 equal to the remaining four is your answers. III. (2x + y)2 − 2xy (i) 36 × 15 ÷ 27 × 13 IV. (2x + 3y)2 − 8y(2x + y) (ii) 53 × 4 × 64 ÷ 16 × 7 (a) I and II only (iii) 328 ÷ 41 × 21 + 9 × 23 (b) I, II and III only (c) II and IV only (iv) 1024 × 11 − 16 × 7 (d) I, II and IV only (v) 17 × 18 − 121 × 6 (e) None of these (a) I (b) II 12. Salary of an officer increases (c) III (d) IV every year by 20%. His salary in the year 2001 was Rs. 26,640. (e) V What was his salary in 1999? 9. A shopkeeper sold an article for (a) Rs. 20,000 (b) Rs. 19,028 Rs. 6,750 after given a discount of 10% on the labelled price. He (c) Rs. 18,500 (d) Rs. 18,840 would have earned a profit of (e) None of these 50%, had there been no discount. What was the actual 13. What approximate value percentage of profit earned ? should come in place of the question mark (?) in the (a) 36 (b) 40 following equation ? (c) 35 95.9753.5 ÷ 16.0013.5 × 6.0021.5 ÷ (d) Cannot be determined 35.992 = ? (e) None of these (a) 36 (b) 16 10. From a group of 7 men and 6 (c) 96 (d) 32 women 5 persons are to be (e) 6 selected to form a committee so that at least 3 men are there on 14. Mr. Anand deposited a total the committee. In how many amount of Rs. 65,000 in three different ways can it be done ? different schemes A, B and C with rates of interest 12 p.c.p.a., (a) 756 (b) 735 16 p.c.p.a. and 18 p.c.p.a.
  • 4. Practice Paper (Solved) ] [ QA – 4 respectively and earned a total (a) 81 litres (b) 72 litres interest of Rs. 10,180 in one (c) 54 litres (d) 66 litres year. If the amount invested in Scheme A was 72% of the 1 1 18. If p + p− 2 amount invested in Scheme 'C', 2 = a, and = b then p p what was the amount invested which of the following is in Scheme B ? correctly expressed ? (a) Rs. 25,000 (a) a − b2 − 2 = 0 (b) Rs. 22,000 (b) a2 + b = 2 (c) Rs. 18,000 (c) a2 − b2 = 1 (d) a2 = b2 (d) Cannot be determined 19. If a + b + c = 0, then the value of (e) None of these a(c + a) (a + b) − b(a + b)(b + c) is 15. In how many different ways can equal to : the letters of the word TRAINER be arranged so that the vowels (a) 1 (b) a b c a b always come together? + c (c) (d) 0 (a) 1440 (b) 120 (c) 720 (d) 360 20. A worker earns a 5% raise. A year later, the worker receives (e) None of these a 2.5% cut in pay, and now his 16. What will be the value of : salary is Rs. 22702.68. What was his salary to begin with ? 98 − 72 + 50 (a) Rs. 22000 (b) Rs. 22176 18 38 (c) Rs. 25000 (d) Rs. 22193 (a) 6 (b) 4 3 21. What will come in place of the (c) − 3 (d) 2 question mark (?) in the following series ? 17. In a hotel, there are dishonest waiters. One of them takes out 2 3 10 39 172 ? one third of the milk from a (a) 704 (b) 885 container full of milk and replaces it with equal quantity (c) 785 (d) 804 of water. A second waiter again (e) None of these takes out one third of the 22. A man received a cheque in mixture and replaces it with which the rupees were equal quantity of water. The transposed for paise and vice process is repeated by 4 waiters versa. After spending 5 rupees resulting in only 16 litres of 42 paise, he discovered that he milk being left in the container. now had exactly six times the What is the capacity of the value of the correct cheque container ?
  • 5. QA – 5 ] [ Practice Paper amount. What amount should 27. Monthly incomes of two he have received ? persons are in the ratio 5 : 4 and their monthly expenditures are (a) Rs. 6.44 (b) Rs. 3.22 in the ratio of 9 : 7. If each (c) Rs. 18.25 (d) Rs. 8.36 person saves Rs. 500 per month, 23. If α and β are the roots of the then what are their monthly quadratic equation ax2 + bx + c incomes ? = 0, then the value of (a) Rs. 8000 and Rs. 10000 2 2 α β (b) Rs. 3750 and Rs. 3000 + is β α 3bc a 3abc b (c) Rs. 5000 and Rs. 4000 − 3 − 3 (d) None of these (a) b c (b) a c 2 2 28. Five persons A, B, C, D and E 3abc b ab b − 2 − 2c occupy seats in a row such that (c) a c (d) 2b 2 c A and B sit next to each other. 3 In how many possible ways can 24. 3 chairs and 2 tables cost Rs. these five people sit ? 700, while 5 chairs and 3 tables (a) 24 (b) 48 cost Rs. 1100. What is the cost of 2 chairs and 2 tables ? (c) 72 (d) None of these (a) Rs. 300 (b) Rs. 350 29. 26 × 12 ÷ 8 + ? = 76 (c) Rs. 450 (d) Rs. 600 (a) 39 (b) 42 25. If a, b are the two roots of a (c) 43 (d) 37 quadratic equation such that (e) None of these a + b = 24 and a − b = 8, then the quadratic equation having a 30. The MSEB electricity bills are and b as its roots is calculated in the following manner. The change in meter (a) x2 + 2x + 8 = 0 reading for the month is (b) x2 − 4x + 8 = 0 rounded off to the next highest multiple of 10. The result is (c) x2 − 24x + 128 = 0 multiplied by 55 paise, and the (d) 2x2 + 8x + 9 = 0 sum is rounded off to the next rupee. If the reading last month 26. A sum of money was divided was 17385 units and this month among two persons x and y in it is 18293 units, what is the bill the ratio 4 : 5. x received Rs. 5 for this month ? less than y. What is the total amount of money : (a) Rs. 501 (b) Rs. 495 (a) 45 (b) 50 (c) Rs. 500 (d) Rs. 505 (c) 90 (d) 250 31. A gambler pays Rs. 3 and gets to throw a dice. He receives an
  • 6. Practice Paper (Solved) ] [ QA – 6 amount equal to the number (c) 100 litres (d) 120 litres that the top face of the dice 37. There are 10 pairs of socks in a shows. If the gambler keeps on drawer. What is the minimum playing the game, how much number of socks that a person does he win per throw, in the should pull out from the drawer long run ? ensure that he gets at least 2 (a) 50 Ps. (b) Rs. 1 matching pairs of socks ? (c) −50 Ps. (d) Rs. 0 (a) 12 (b) 11 − − 32. |P−10| = 12 & |4J− 10| = 6. What (c) 5 (d) 10 P 38. A painting show drew crowds is maximum value of . which doubled in number each J day. If the show opened on (a) −11 (b) 22 Monday and the number of 11 spectators on Saturday was (c) −2 (d) 2 6400, what was the number on 33. If x + y > 5 and x − y > 3, then the opening day? which of the following gives all (a) 100 (b) 200 possible values of x ? (c) 800 (d) 80 (a) x > 3 (b) x > 4 39. 15 chairs and 2 tables cost Rs. (c) x > 5 (d) x < 5 4,000. Find the cost of 12 chairs 34. 136 × 25 ÷ 16 × ? = 2550 and 2 tables, if the cost of 10 chairs be equal to that of 5 (a) 12 (b) 22 tables. (c) 20 (d) 18 (a) Rs. 4,000 (b) Rs. 4,200 (e) None of these (c) Rs. 3,900 (d) Rs. 3,600 35. The probability of rain on day 40. 5 chairs and 2 tables cost Rs. 1 is 0.2 and the probability on 1,080. The cost of 2 chairs is day 2 is 0.3. What is the equal to that of a table. Find the probability of raining on both cost of 2 chairs and 5 tables. the days? (a) Rs. 1,440 (b) Rs. 1,480 (a) 0.2 (b) 0.1 (c) Rs. 1,380 (d) Rs. 1,420 (c) 0.06 (d) 0.25 41. An employee spends 30% of his 36. Three − fourths of a tank is full salary on food and donates 3% of water. If 5 litres are added to of his salary. If he spends Rs. − it then four−fifths of the tank 231 on these two items what is becomes full. What is the his salary ? capacity of the tank? (a) Rs. 1250 (b) Rs. 700 (a) 75 litres (b) 80 litres (c) Rs. 630 (d) Rs. 940
  • 7. QA – 7 ] [ Practice Paper 23 1 42. The mean annual salary paid to all the staff members of a (c) 30 (d) 2 company was Rs. 5000. The 47. A student on his birthday mean annual salary paid to distributed on an average 5 male and female staff were Rs. chocolates per student. If on the 5200 and Rs. 4500 respectively. arrival of the teacher and the Determine the number of male headmaster to whom the and female staff members of the student gives 10 and 15 company. chocolates respectively, the (a) 80, 20 (b) 70, 30 average chocolate distributed (c) 60, 40 (d) 40, 60 per head increases to 5.5, then what is the strength of the 43. A man has some hens & cows. class? If the number of heads be 48 and the number of feet equals 140, (a) 28 (b) 30 the number of hens will be (c) 32 (d) None of these (a) 26 (b) 24 48. Find the value of (c) 23 (d) 22 0.2 × 0.2 × 0.2 + 0.02 × 0.02 × 0.02 2 0.4 × 0.4 × 0.4 + 0.04 × 0.04 × 0.04 44. If a b = 5b + a , then (a, b) could be 0.67 × 0.67 × 0.67 − 0.001 + 0.67 × 0.67 + 0.067 + 0.01 (a) (3, 4) (b) (2, 12) (a) 4.87 (b) 1.07 (c) (4, 18) (d) (6, 4) (c) 0.067 (d) 0.002 45. How many bricks are required to build a wall of 15 metres 49. Find the value of length, 12 metres height and 20 0.03 × 0.03 + 0.01 × 0.01 - 0.02 × 0.03 cm thickness, if the brick is 36 cm long, 25 cm wide and 10 cm 0.02 thick ? FG 3 + 1 of 3 IJ (a) 2000 (b) 4000 ÷ H 8 2 16K (c) 12000 (d) None of these (a) 0.447 (b) 8.04 46. Three students try to solve a (c) 0.0427 (d) 0.012 problem independently with a probability of solving it as 0.289 24 + 216 50. Simplify : + 2 4 3 0.00121 96 , , respectively. What is 8 5 4 (a) 12.45 (b) 16.54 the probability that the problem is solved ? (c) 18.90 (d) 17.45 12 3 51. A pack of 52 cards is distributed (a) (b) 30 amongst 4 players. The one to 20
  • 8. Practice Paper (Solved) ] [ QA – 8 receive the set with the lowest (c) 9 (d) 12 sum wins 56. If x1/3 + y1/3 + z1/3 = 0, then : (A = 1, J = 11, Q = 12, K = 13). (a) x + y + z = 0 What is the least total with (b) (x + y + z)3 = 27 xyz which one can win ? (c) x + y + z = 3 xyz (a) 40 (b) 31 (d) x3 + y3 + z3 = 0 (c) 28 (d) 24 57. A crown, made of gold, silver, 52. If every 2 out of 3 readymade copper and brass weighs 9.725 shirts need alterations in the kg. The weight of the gold and collar, every 3 out of 4 need silver together is 4 kg and the alterations in the sleeves, and weight of the gold and copper every 4 out of 5 need it in the 4.5 kg and of the gold and brass body, how many alterations 3.6 kg. What is the weight of will be required for 60 shirts ? gold in the crown ? (a) 123 (b) 133 (a) 1.2500 kg (b) 2.6575 kg (c) 143 (d) 24 (c) 1.1875 kg (d) 2.3705 kg 53. Which of the following does not 58. A bag contains 3 white balls belong to the group ? and 2 black balls. Another bag 10 , 11 , 18, 36, 74 contains 2 white balls and 4 black balls. A bag and a ball are (a) 10 (b) 11 picked at random. The (c) 18 (d) 36 probability that the ball will be 54. Tulsi had a children's party and white is bought two mangoes for each 7 7 child. However, a quarter of the (a) 11 (b) 30 kids invited did not come. 25 boys came and the surplus 5 7 provided just one extra mango (c) 11 (d) 15 for each girl. How many mangoes did Tulsi buy ? 59. One hundred identical coins (a) 200 (b) 132 each with probability p of showing up heads are tossed. If (c) 150 (d) 128 0 < p < 1 and the probability of − 55. If x is a three−digit number and heads showing on 50 coins is y is a number obtained by equal to that of heads on 51 permuting the digits of x in any coins, then the value of p is manner, then (x − y) is always 1 49 divisible by : (a) 2 (b) 101 (a) 4 (b) 6
  • 9. QA – 9 ] [ Practice Paper 50 51 65. With the same data as for the (c) 101 (d) 101 previous question, what is the probability that the next 60. Two dice are tossed. The program will run correctly probability that the total score after the third run, but not is a prime number is earlier ? 9 3 1 5 (a) 10 (b) 10 (a) 6 (b) 12 7 1 1 7 (c) 20 (d) 10 (c) 2 (d) 9 66. A number is greater than the FG 1 IJ −2 square of 44 but smaller than 61. The value of H 4K 1 is the square of 45. If one part of the number is the square of 6 and the number is the multiple (a) 2 (b) − 2 of 5, then find the number. 1 (a) 1940 (b) 2080 (c) − 16 (d) 16 (c) 1980 62. The volume of a cube is V. The total length of its edges is (d) Cannot be determined (e) None of these (a) 6 V1/3 (b) 8 V 67. Rajesh solved 80 per cent of the (c) 12 V2/3 (d) 12 V1/3 questions in an examination. If 63. A boy was asked to write 25 × out of 41 questions solved by 9 2 , but he wrote 2592. The Rajesh 37 questions are correct numerical difference between and of the remaining questions the two is out of 8 questions 5 questions have been solved by Rajesh (a) 0 (b) 1 correctly then find the total (c) 2 (d) 3 number of questions asked in the examination ? x 2 − 36 x+6 64. If P = and Q = , then (a) 75 (b) 65 x − 49 2 x+7 P (c) 60 the value of is Q (d) Cannot be determined x 6 x 6 −7 − (e) None of these (a) x (b) x 7 − + 68. What was the day on April 6, x 2001? (c) x −76 x+6 (a) Friday (b) Saturday + (d) x − 7 (c) Thursday (d) Sunday
  • 10. Practice Paper (Solved) ] [ QA – 10 69. Monthly incomes of two 74. 76.59 + 129.052 − 38.314 = ? + persons are in the ratio 5 : 4 and 45.72 their monthly expenditures are in the ratio of 9 : 7. If each (a) 121.068 (b) 121.608 person saves Rs. 500 per month, (c) 120.068 (d) 120.608 then what are their monthly incomes ? (e) None of these (a) Rs. 8000 and Rs. 10000 75. 336 ÷ 12 × 15 − ? = 138 (b) Rs. 3750 and Rs. 3000 (a) 140 (b) 233 (c) Rs. 5000 and Rs. 4000 (c) 420 (d) 282 (d) None of these (d) None of these 76. 168 × 15 ÷ 24 × 12 = ? 70. Income of C is 20% more than that of B and income of B is 25% (a) 1160 (b) 8.75 more than that of A. Find out (c) 1260 (d) 105 by how much % is the income of C more than that of A. (e) None of these (a) 25% (b) 75% 77. 4410 ÷ 45 ÷ 7 = ? (c) 50% (d)100% (a) 98 (b) 686 71. Spending Rs. 1,200 daily for 7 (c) 14 (d) 70 days, I ran into a debt which (e) None of these was cleared in 9 days after I reduced my daily expense to Rs. 78. 7586 + 11254 − ? = 8976 880. Find my daily income. (a) 9846 (b) 9764 (a) Rs. 1,000 (b) Rs. 1,020 (c) 9784 (d) 9864 (c) Rs. 1,040 (d) Rs. 1,025 (e) None of these 72. The ratio of incomes of A and B 79. 1111 + 12121 + 1020102 = ? is 5 : 3 and that their expenditures is 8 : 5. If their (a) 1303334 (b) 1033344 savings are in the ratio of 2 : 1 (c) 103334 (d) 1033334 and their total saving is Rs. 3,600, then the income of A is : (e) None of these (a) Rs. 12,000 (b) Rs. 7,200 80. There are two grades A and B of workers in a workshop. (c) Rs. 7,800 (d) Rs. 9,100 Every worker contributes as many rupees as there are 73. 1.5 × 1.2 − 0.06 × 0.5 = ? workers of his own category. If (a) 1.77 (b) 17.97 the total amount contributed is (c) 1.797 (d) 17.77 Rs. 196 including Rs. 16 contributed by the owner of the (e) None of these workshop, what is the total
  • 11. QA – 11 ] [ Practice Paper number of workers in that 88. A person wants to divide a sum workshop ? of Rs. 3,90,300 between his two sons who are 13 and 15 years of (a) 18 (b) 14 (c) 12 age respectively in such a way (d) 10 (e) None of these that their shares, if invested at 4% per annum compound 196 17 78 81. × × =? interest, should produce the 14 289 169 same amount when they (a) 1 (b) 2 (c) 6 become 18 years of age. Find the (d) 4 (e) 13 share of each. ? (a) 187500, 202800 82. = 250 .25 (b) 178500, 183000 (a) 500 (b) 125 (c) 5 (c) 199400, 194500 (d) 0 (e) 100 (d) 168390, 195600 189 89. A, B and C invested Rs. 26,000, 83. = 1.89 Rs. 34,000 and Rs. 10,000 a respectively in a business. (a) 10 (b) 100 (c) 1000 They earn a profit of Rs. 3500. (d) 10000 (e) None of these B's share of profit is (a) Rs. 1200 (b) Rs. 1500 84. 12 + 24 equals (c) Rs. 1700 (d) Rs. 1900 (a) 2 6 + 2 3 (b) 36 90. Mr. Rai decided to distribute his (c) 288 (d) 6 2 income among the members of 85. Find 100 + 49 his family. He gave 50% to his wife, 35% of the remaining to (a) 149 (b) 17 (c) 490 both of his sons, and the (d) balance of Rs. 6,750 was 14 + 10 (e) None of these deposited by him in the bank. 86. 0.00004761 equals How much amount was received by his wife ? (a) 0.069 (b) 0.0069 (a) Rs. 23,200 (b) Rs. 45,000 (c) 0.00069 (d) 0.0609 (c) Rs. 22,500 (d) Rs. 13,500 3 87. If x = , find the value of (e) None of these 2 91. If a ⊗ b = (a × b) + b, then 5 ⊗ 7 1 + x+ 1 - x equals to 1 + x− 1 - x (a) 12 (b) 35 (a) 5 (b) 3 (c) 42 (d) 50 (c) 2 (d) 4 92. 24% of 250 + ?% of 240 = 120
  • 12. Practice Paper (Solved) ] [ QA – 12 (a) 25 (b) 40 98. (1502)2 − (1498)2 = ? (c) 30 (d) 45 (a) 12,000 (b) 16,000 (e) None of these (c) 22,56,004 (d) 22,560 93. 22% of ? + 30% of 420 = 192 3 99. of 480 ÷ 8 + 82 = ? (a) 330 (b) 350 5 (a) 120 (b) 100 (c) 200 (d) 280 (c) 36 (d) 44 (e) None of these 100. Value of 64 ÷ 8 ÷ 4 ÷ 2 is 94. 75 × 18 + ?% of 150 = 1380 (a) 1 (b) 8 (a) 25 (b) 20 (c) 16 (d) 24 (c) 12 (d) 16 (e) None of these (e) None of these 101. If a : b = 2 : 3, b : c = 5 : 7, then a 95. Subhash bought 20 kg of tea at : b : c is the rate of Rs. 30 per kg and 30 kg at the rate of Rs. 25 per kg. (a) 2 : 3 : 7 (b) 2 : 5 : 7 He mixed the two and sold the (c) 10 : 15 : 21 (d) 2 : 15 : 7 mixture at the rate of Rs. 22.50 per kg. What was his loss in the (e) None of these transaction ? 102. If two numbers are in the ratio (a) Rs. 200 (b) Rs. 225 5 : 7and their least common multiple is 315, then their (c) Rs. 175 (d) Rs. 200.25 product is 96. A boy goes to school with the (a) 2385 (b) 2538 speed of 3 km/hr and returns with a speed of 2 km/hr. If he (c) 2358 (d) 2835 takes 5 hours in all, the distance 103. If x : y = 3 : 4, y : z = 5 : 6 and z : w in kms between the village and = 2 : 3, then x : w equals the school is (a) 5 : 3 (b) 3 : 3 (a) 6 (b) 7 (c) 5 : 12 (d) 7 : 3 (c) 8 (d) 9 104. Subtract − 13 from 28 − 5 + 5. 97. Gold is 19 times heavy as water (a) 51 (b) 53 and copper 9 times as heavy as water. The ratio in which these (c) 56 (d) 58 two metals be mixed so that the (e) None of these mixed so that the mixture is 15 times as heavy as water, is 105. Add 7.007, 70.7 and 7.007 (a) 1 : 2 (b) 2 : 3 (a) 84.074 (b) 84.714 (c) 3 : 2 (d) 19 : 135 (c) 84.741 (d) 80.714
  • 13. QA – 13 ] [ Practice Paper 106. Which of the following 2 1 112. Express of of Rs. 25.20 as a numbers is a multiple of 8 ? 3 4 (a) 923862 (b) 923962 1 fraction of 1 of Rs. 36. 2 (c) 923972 (d) 923872 7 11 107. If 14% of a number is 105, then (a) (b) 90 90 find the number. 5 5 (a) 715 (b) 705 (c) (d) 8 42 (c) 735 (d) 750 113. Simplify : LM R F I UOP 108. Three pipes fill a tank separately in 10 minutes, 20 1+ 2 3− 1+ 2− | G S H 1 5 − JK |PQ V minutes and 30 minutes respectively. An outlet pipe can MN | T 2 2 | W empty it in 15 minutes when no (a) 2 (b) 1 water flows in. If all the pipes (c) −3 (d) 5 are opened, when the tank is empty, then how long, in 114. The least of the following minutes, will it take to fill the tank ? 2 b g 0.2, .2 , 0.2 , 1 ÷ 0.2 is 1 4 (a) (.2)2 (b) 0.2 (a) 9 (b) 8 7 7 (c) 0.2 (d) 1 ÷ 0.2 1 2 (c) 7 (d) 6 115. 5% of 5% of Rs. 100 is 2 3 109. A number is multiplied by its (a) Re. 0.25 (b) Re. 0.50 − one−third to get 192. Find the (c) Rs. 10 (d) Rs. 25 number. 116. The value of (a) 16 (b) 20 (c) 24 (d) 28 6 + 6 + 6 + 6+.... is 110. The value of (a) 2 (b) 5 0.125 + 0.027 is (c) 4 (d) 3 0.5 × 0.5 + 0.09 − 0.15 117. A cistern is normally filled with (a) 1 (b) 0.2 water in 10 hours but takes 5 (c) 0.08 (d) 0.8 hours longer to fill because of a leak in its bottom. If the 0.01 + 0.0064 cistern is full, then the leak will 111. Evaluate : 0.01 × 0.3 empty the cistern in (a) 1 (b) 10 (a) 20 hours (b) 40 hours (c) 100 (d) 1000 (c) 50 hours (d) 30 hours
  • 14. Practice Paper (Solved) ] [ QA – 14 118. Taps A and B can fill a bucket in 12 minutes and 15 minutes b12.12g − b8.12g 2 2 respectively. If both are opened and A is closed after 3 minutes, 120. Simplify b0.25g + b0.25gb19.99g 2 how much further time would LMe8 j OP × 16 -3/4 5/2 8/15 3/4 +N Q it take for B to fill the bucket ? (a) 8 min 5 sec. (b) 8 min 15 sec. LMF b128g I OP 3/7 −1/5 NH K Q -5 3 (c) 7 min 45 sec. (d) 7 min 15 sec. 119. A and B enter into a 1 1 partnership investing Rs. (a) 4 (b) 6 12,000 and Rs. 16,000 2 2 respectively. After 8 months, C 2 1 also joins the business with a (c) 3 (d) 8 3 2 capital of Rs. 15,000. The share 121. Find the value of of C in a profit of Rs. 45,600 after 1 1 e28 + 10 3 j − e7 − 4 3 j two years is 2 2 (a) Rs. 12,000 (b) Rs. 14,400 (a) 5 (b) 3 (c) Rs. 19,200 (d) Rs. 21,200 (c) 8 (d) 6 (e) None of these ANSWERS & SOLUTIONS Ans.1.(a) Let three students are A, B and Ans.3.(c) There are two ways of C ⇒ Probability that the given problem borrowing books cannot be solved by A, B & C is (a) When there is no chemistry book in 1 2 5 2 3 7 7 FG 1 IJ FG 1 1IJ FG IJ this case, 3 books are to be selected from H −3 KH −5 KH K − 12 = 3 × 5 × 12 = 30 the remaining 6 books : ⇒ Probability that the problem is solved This can be done in = 6C3 7 FG IJ 30 = 23 6 5 4 × × =1− H K30 = 1 2 3 = 20 ways × × Ans.2.(a) One principal can be (b) When there is a chemistry 1 and appointed out of 36 teachers = 36C1, 36 chemistry 11 books. In this case one ways. book can be selected from the remaining For each way of doing so, one vice 6 books in = 6C1 = 6 ways. Hence, total number of ways = 20 + 6 = 26 principal can be appointed out of the remaining 35 teachers in 35C1 = 35 ways Ans.4.(d) The equation is same only if k=6 Hence, two posts, together can be filled (by multiplying the first equation by 2). in 36 × 35 = 1260 ways.
  • 15. QA – 15 ] [ Practice Paper Ans.5.(a) Probability of not given Ans.13.(e) Given expression 5 8 7 20 5 + + 96 3 × 96 6× 6 5 8 7 4 = 24 = 6 = × 3 + + + 16 × 16 64 Ans.6.(b) 7! = (3C2 × 6! × 2! + 5!) simplyfying the expression, = 5040 − (3 × 720 × 2 + 120) = 1440. we get its value as 6. Ans.7.(b) In the second year the Ans.14.(b) If C = 100, A = 72. investment are : Hence ratio = 25 : 18. 5 × 1.26 : 7 × 1.20 : 6 × 1.15 Then 18x + 25x + y = 65,000 and 12% = 6.3 : 8.4 : 6.9 = 21 : 28 : 23. (18x) + 18% (25x) + 16% (y) = 10, 180. 15 Solving the two equations, Ans.8.(a) 36 × 27 × 13 = 260. we get y = 22,000. All other parts equal 240. Ans.15.(c) Country the vowels as one, 100 Ans.9.(c) MP = 6750 × 100 we have 5! ways. 90 = 7500. Since the vowels can be arranged in 3! × 150 . Since profit1750 = 50%, C.P. = 7500 ways, the reqd. answer is 5! × 3! = 720. Profit % = 5000 × 100 = 35%. Ans.16.(d) Reduce to the base of 2 by Ans.10.(a) Since there are at least 3 factorisation and solve. men in the committee, we can have 3 Ans.17.(a) Milk left after 4th operation/ cases, either 3 or 4 or 5 men. Whole quantity of container 1 4 16 (i) 7C3 × 6C3 (ii) 7C4 × 6C1 1 FG IJ (iii) 7C 5 × 6C 0 = 525 + 210 + 21 = 756 = H −3 = x K ⇒ x = 81 litres. Ans.11.(c) p 1 2 FG IJ 1 − p = P 2 + 2 − 2. I. 9x2 + y2 + 6xy − 5x2 + 2xy Ans.18.(a) H K P = 4x2 + y2 + 8xy Hence b2 = a − 2. II. (2x − y)2 = 4x2 + y2 − 4xy Ans.19.(d) III. 4x2 + y2 + 4xy − 2xy a(c + a)(a + b) − b (a +b)(b + c) = 4x2 + y2 + 2xy = abc − abc = 0. IV. 4x2 + 9y2 + 12xy − 16xy − 8y2 Ans.20.(b) Suppose the salary was Rs. 100, to begin with. 3= 4x2 + y2 − 4xy ⇒ 100 + 5% = 105 Hence only II and IV are equal. ⇒ 105 − 2.5% = 102.375 Ans.12.(c) Salary 2 years back If the present salary is Rs. 102.375, then 100 100 the salary in the beginning was Rs. 100. = 26,640 × × 120 120 = 18,500.
  • 16. Practice Paper (Solved) ] [ QA – 16 If the present salary Rs. 22702.68, then Ans. 25.(c) a + b = 24, a − b = 8 the salary the beginning was 100 ⇒ a = 16, b = 8 ⇒ ab = 128 102.375 × 22702.68 = 22176. ⇒ Required equation is the one whose sum of the roots is 24 and product of the Ans.21.(b) The sequence in the given roots is 128. series is i.e. x2 − 24 x + 128 = 0 × 1 + 12, × 2 + 22, × 3 + 32, × 4 + 42, x × 5 + 52. y =4 Ans.26.(a) 5,x=y−5 Ans.22.(a) The cheque was received for 4y 4y Rs. 44.06. After spending Rs. 5.42, he ⇒ x= 5 , 5 =y−5 had Rs. 38.64, which is 6 times of Rs. 6.44. ⇒ y = 25, x = 20 ⇒ x + y = 45 ∴ He should have received Rs. 6.44. = Total amount c b Ans.23.(b) α + β = − αβ = a Ans.27.(c) Let the monthly incomes of a, two persons be 5I and 4I and their α2 + β2 = (α + β)2 − 2αβ monthly expenditures be 9E and 7E b c b ac respectively. 2 2 2 −2 = a − a = a 2 2 ⇒ 5I − 9E = 500 α 2 β2 α 3 + β 3 ⇒ 4I − 7E = 500 ⇒ + = β α αβ ⇒ I = 1000, E = 500 (α + β)(α 2 + β2 − αβ) ⇒ Monthly incomes of the two persons = are Rs. 5000 and Rs. 4000. αβ FG − b IJ FG b 2 − 2ac c I JK Ans.28.(b) 4! × 2 ways, = H aKH a2 − a i.e. 24 × 2 = 48 ways. c Ans.29.(d) 26 × 12 ÷ 8 + ? = 76 a 2 3 12 − b (b − 3 a c) a abc − b 3 ⇒ 26 × + ? = 76 3 × = 8 a c a 2c ⇒ ? = 76 − 39 = 37 Ans.24.(d) 3C +2T = 700 Ans.30.(a) Change in meter reading 5C + 3T = 1100 = 18293 − 17385 = 908 units. ⇒ 9C + 6T = 2100 Rounded off to 910 units. ⇒ 10C + 6T = 2200 Therefore Bill is (910)(55) = 50050 paise, ⇒ C = 100, T = 200 or Rs. 500.50. ⇒ 2C + 2T = 200 + 400 = 600. This is rounded off to Rs. 501.
  • 17. QA – 17 ] [ Practice Paper 10 Ans.31.(a) In the long run, every number will have occurred roughly the Ans.37.(c) x (5) 2 = 5 same number of times. Ans.38.(b) Let the no. of crowds on the Therefore for every 6 throws, each opening day = x number from 1 to 6 would have occurred. Monday = x, Tuesday = 2x Therefore the gambler would have won 1 + 2 + 3 + 4 + 5 + 6 i.e. Rs. 21. Wednesday = 4x, Thursday = 8x He would have spent Rs. (6 × 3) = 18. Friday = 16x, Therefore gain is Rs. 3 in 6 throws, Saturday = 32x = 6400 i.e. 50 paise per throw. 6400 P ⇒ x = 32 = 200. Ans.32.(b) J max ⇒ P max, J min Ans.39.(c) Let cost of a chair and a table [P − 10] = 12 ⇒ P = 22 or − 2 be Rs. x and y respectively. [4J − 10] = 6 ⇒ J = 4 or 1. Then, 15x + 2y = 4000 .....(i) P 10x = 4y ....(ii) So max. value of J is 22. Solving (i) and (ii) Ans.33.(b) x + y > 5 x = 200, y = 500 x − y > 3, 2x > 8, x > 4. 12x + 3y = 12 × 200 + 3 × 500 Ans.34.(a) 136 × 25 ÷ 16 × ? = 2550 = Rs. 3900 25 Ans.40.(a) 5x + 2y = 1080 .....(i) ⇒ 136 × × ? = 2550 16 2x = y or 2x − y = 0 ......(ii) 2550 × 16 Solving (i) and (ii), ⇒ ? = 136 25 = 12 × x = 120, y = 240. Ans.35.(c) P(A) = 0.2, P(B) = 0.3 ⇒ 2x + 5y = 2 × 120 + 5 × 240 P(A ∩ B) = P(A).P(B) = Rs. 1440. = 0.2 × 0.3 = 0.06 Ans.41.(b) Let his salary = Rs. x Ans.36.(c) Expenditure on food Let the capacity of tank = x liter. 30 3x 3 4 3x 4 x =x × 100 = Rs. 10 4 x + 5 = 5 x ⇒ 4 − 5 = −5 3 3x x 15x 16x Donation = × 100 = 100 − 20 −x = −5 ⇒ −5 3x 3x 20 = ⇒ 10 + 100 = 231 ⇒ x = 100
  • 18. Practice Paper (Solved) ] [ QA – 18 30x 3 x 1 4 3 12 3 100+ 4 × 5 × 4 = 80 = 20 ⇒ = 231 Ans.46.(a) 33 x Ans.47.(a) Suppose strength of the class ⇒ 100 = 231 = x ⇒ 5x + 10 + 15 = 5.5(x + 2) ⇒ 33x = 231 × 100 100 ⇒ 0.5x = 14 ⇒ x = 28 ⇒ x = 231 × 33 = 700. Ans.48.(b) Given expression 1 (0.67) 3 − (0.1) 3 Ans.42.(b) Let the of ratio of male and = + female is K : 1 2 (0.67) 2 + 0.67 × 0.1 + (0.1) 2 1 ⇒ 5200 K + 4500 = (K + 1) × 5000 = 2 + (0.67 − 0.1) ⇒ 5200 K + 4500 = 5000K + 5000 [ ... a3 − b3 = (a − b)(a2 + b2 + ab)] ⇒ 5200 K − 5000K = 5000 − 4500 = 500 = 0.5 + 0.57 = 1.07 500 5 Ans.49.(c) Given expression 200 K = 500 ⇒ K = 200 = 2 (0.03) 0.01 2 2 0.01 0.03 5 = 2 + b g − × 0.02 × K:1= 2 :1=5:2 FG 3 + 3 IJ = 70 : 30 = 70, 30 ÷ H 8 32 K Ans.43.(a) Let No. of hens = x (0.03 − 0.01) 2 32 0.02 × 32 × = ⇒ No. of cows = (48 − x) = 0.02 15 15 ⇒ 2x + 4 (48 − x) = 140 0.64 64 16 = = = 0.0427 15 1500 375 = ⇒ 2x + 192 − 4x = 140 Ans.50.(d) Given expression ⇒ −2x = 140 − 192 = −52 52 28900000 24 + 24 × 9 = + ⇒ x = 2 = 26 121000 96 Ans.44.(b) Put a = 2, b = 12 in 170 4 24 170 192 = + = +2= = 17.45 a 11 2 24 11 11 b = 5b + a2 Ans.51.(c) Required number 2 ⇒ 12 = 5 × 12 + 4 = 64 = (4 × 1) + (4 × 2) + (4 × 3) + (1 × 4) ⇒ 26 = 64, which is true. = 4 + 8 + 12 + 4 = 28. Ans.45.(b) No. of bricks required Ans.52.(b) Total number of alteration 1500 1200 20 for 60 shirts 2 3 4 = 36 × 25 70 × = 4000 × × = [ 3 × 60 + 4 × 60 + 5 × 60] = 133.
  • 19. QA – 19 ] [ Practice Paper Ans.53.(d) 10 = 10 + 03 i.e. G + S + C + B = 9.725 ......(i) ⇒ 11 = 10 + 13 ⇒ 18 = 10 + 23 where G = part of gold in crown S = part of silver in crown ⇒ 36 = 10 + ?3 C = part of copper in crown ⇒ 74 = 10 + 43 B = part of brass in crown The number in the series at position 4 must be 10 + 33 = 37. Again by the condition of the question : Therefore the number 36 does not G + S = 4 kg .......(ii) belong to the group. G + C = 4.5 kg .......(iii) Ans.54.(a) Let number of children = x. G + B = 3.6 kg .......(iv) So number of mangoes purchased = 2x. add (ii) + (iii) + (iv) Number of children present x 3x 3G + S + C + B = 12.1 kg =x− 4 = 4 . 2G + 9.725 = 12.1 ⇒ 2G = 2.375 3x Number of girls = 4 − 25. ⇒ G = 1.1875 kg. Now by the question, Ans.58.(d) Prob. that bag A is drawn = 1 FG 3x − 25IJ × 3 + a25 × 2f = 2x 2 . Prob. that white ball is drawn from 1 H4 K bag A = 2 × = 3 5 10 3 ⇒ x = 100. So the number of mangoes 1 purchased = 200. Prob. that bag B is drawn = 2 Ans.55.(c) Suppose the hundred's, ten's Prob. that white ball is drawn from bag and unit's places of x be a, b, c 1 2 1 × = . respectively. Make these digits as c, b, a. B= 2 6 6 Difference of numbers Prob. that white ball is drawn either = (100a + 10b + c) − (100c + 10b + a) from bag A or from bag B = 99 (a − c), which is divisible by 9 but 3 1 7 = + = none of 4, 6, 12. 10 6 15 Ans.56.(b) Using a + b + c = 0 Ans.59.(a) P50 (1 − p)50 = P51 (1−P)49 1 ⇒ a3 + b3 + c3 = 3abc, ⇒ P= 2 we get, x1/3 + y1/3 + z1/3 = 0 Ans.60.(b) Total score will be a prime ⇒ x + y + z = 3x1/3 y1/3 z1/3 number in 15 ways out of 36 : ⇒ (x + y + z)3 = 27 xyz. (1,1), (1,2), (1,4), (1,6), (2,1), Ans.57.(c) Given that the crown made (2,3), (2,5), (3,2), (3,4), (4,1), of gold, silver, copper and brass and (4,3), (5,2), (5,6), (6,1), (6,5). weighs 9.725 kg,
  • 20. Practice Paper (Solved) ] [ QA – 20 4 Hence, the required probability = 80% of n = 5 n 15 5 = = 36 12 Now, according to the question, FG 1 IJ −2 FG 4 IJ 2 5 4n Ans.61.(d) H 4K = H 1K = 16 37 + (n − 4) × 5n 8 = 5 205 4n Ans.62.(d) There are 12 edges in the cube, Volume = V. or, 37 + 8 − 8 = 5 205 4n 5n Each edge = V1/3. or, 37 − 8 = 5 − 8 Total length of the edges = 12 V1/3. 296 205 32 25 Ans.63.(a) 25 × 92 = 32 × 81 = 2592 − 8 − n40 n or, = P x 2 − 36 x + 7 x−6 Ans.64.(a) = 2 × = . Q x − 49 x + 6 x −7 91 7n 2 7 5 = 1 FG 20 + 20 + 20 = 3 IJ or 8 91 40 40 Ans.65.(b) − H 10 K × ⇒ n = 8 7 = 65 × Ans.66.(c) Let the number be N Therefore, total number of questions = ⇒ According to the question, 65. Thus the required answer is option (b). (44)2 < N < (45)2 ⇒ 1935 < N < 2025 Ans.68.(a) Complete years 2000. No. of Therefore, the required number would odd−days in 2000 = 0. be any number between 1937 and 2025. Odd day in Jan = 3, Feb = 0, March = 3, But from the question it is clear that the April 6, Total = 12, and odd days = 5. required number is the factor of 6 and Day = Sunday + 5 = Friday the multiple of 5. So we have to find out the number between 1937 and 2025 Ans.69.(c) Let x denote the monthly which is divisible by both 36 and 5. income variable and y denote the monthly expenditure. 62 = 36 As per question, LCM of 36 and 5 = 36 × 5 = 180 5 9 180 × 10 = 1800 x− y = 500 .................(i) 9 16 180 × 11 = 1980 4 7 Thus the required no. is 1980. and x− y = 500 .........(ii) 9 16 So, Answer is (c). Solving (i) and (ii) x = 900 5 Ans.67.(b) Let the total number of questions asked in examination be n. ⇒ Their monthly income are 9 x and 4 ⇒ No. of correct answer 9 x = Rs. 5000 and Rs. 4000 respectively.
  • 21. QA – 21 ] [ Practice Paper Ans.70.(c) C = B + 20% of B B 6B ⇒ 13232 + 1020102 = ? =B+ 5 = 5 . ⇒ ? = 1033334 A 5A Ans.80.(a) The contribution by the B = A + 25% of A = A + 4 = 4 workers in the workshop = 196 − 16 = 6 6 5 3 Rs. 180. Let the number of workers in A ⇒ C= 5B= 5×4 A= 2 A grade be x and in B grade be y. A ∴x2 + y2 = 180. = A + 2 = A + 50% of A Now, by putting x = 12 and y = 6, Ans.71.(b) Let my daily income be Rs. we get x2 + y2 = 180 x. ∴x + y = 12 + 6 = 18 Then, 7 (1200 − x) = 9 (x − 880) x = 1020. 196 17 78 Ans.72.(a) Ans.81.(c) × × =? 14 289 169 Saving of A = 2400, of B 1200. 14 × 14 17 78 Then, 5x − 8y = 2400 and 3x − 5y = 1200. ⇒ ?= × × 14 17 × 17 13 × 13 Solving, we get x = 2400. Hence 5x = 12,000. 14 17 78 18564 ⇒ ? = 14 × 17 × 13 = 3094 = 6 Ans.73.(a) ? = 1.5 × 1.2 − 0.06 × 0.5 ? = 1.80 − 0.030 = 1.770 = 1.77 Ans.82.(b) = 250 0.25 Ans.74.(b) ⇒ ? = 250 × 0.25 76.59 + 129.052 − 38.314 = ? + 45.72 ⇒ 250 × 0.5 × 0.5 = 250 × 0.5 ⇒ 205.642 − 38.314 − 45.72 = ? 5 ⇒ 167.328 − 45.72 = ? ⇒ ? = 121.608 ⇒ 250 × = 125 10 Ans.75.(d) 336 ÷ 12 × 15 − ? = 138 Ans.83.(d) 189 = 1.89 ⇒ 189 = a a 1.89 ⇒ ? = 336/12 × 15 − 138 18900 = 28 × 15 − 138 = 420 − 138 = 282 ⇒ = a , Squaring both sides 189 15 Ans.76.(c) ? = 168 × × 12 = 1260 (100)2 = ( a )2 ⇒ 10000 = a 24 Ans.77.(c) 4410 ÷ 45 ÷ 7 = 98 ÷ 7 = 14 Ans.84.(a) 12 + 24 Ans.78.(d) 7586 + 11254 − ? = 8976 = 2×2×3 + 2×2×2×3 = 2 3+2 6 ⇒ 18840 − 8976 = ? Ans.85.(b) 100 + 49 ⇒ ? = 9864 ⇒ 10 × 10 + 7 × 7 = 10 + 7= 17 Ans.79.(d) 1111 + 12121 + 1020102 = ?
  • 22. Practice Paper (Solved) ] [ QA – 22 Ans.86.(b) 0.00004761 FG 4 IJ 5 b g FGH1 + 100 IJK 4 3 4761 69 × 69 H x 1+ 10 0 K = 390300 − x = = 100000000 10000 × 10000 676 x = (390300 − x) 69 625 = = 0.0069 10000 ⇒ x = 187500 and y = 390300 − 187500 Ans.87.(b) ⇒ y = 202800 1 + x + 1− x 1 + x + 1− x × Ans.89.(c) Profit of Rs. 3500 is divided 1 + x − 1−x 1 + x + 1− x among A, B and C in the ratio 26000 : = e 1 + x + 1− x j 2 34000 : 10000. i.e. in the ratio 13 : 17 : 5. e 1 + x j e 2 − 1− x j 2 ∴B's share of profit = 17 × 3500 35 1 + x + 1 − x + 2 1 − x2 1 + 1 − x2 = a 1 + x − 1− xfa f = x = Rs. 1700. Ans.90.(c) 15% of the income = Rs. 6750 1 + 1 − 34 1+ 1 ∴ Total income = Rs. 45000 = = 2= 3 3 3 ⇒ Amount given to the wife by Mr. Rai 2 2 = Rs. 22500 Ans.88.(a) Let x and y be the shares of the two sons. Ans.91.(c) 5 ⊗ 7 = (5 × 7) + 7 = 42 ∴x + y = 3,90,300 24 ? Ans.92.(a) × 250 + × 240 = 120 ⇒ y = (390300 − x) 100 100 For the boy of age 13 years, ? × 12 ⇒ 60 + = 120 = 5 years, Rate = 4%, Principal = x 5 ∴Amount after 5 years compounded ⇒? = b 120 − 60 × 5 = 25 g FG 4 IJ 5 12 annually H = x 1+ 100 K .......(i) Ans.93.(e) 22 100 ×?+ 30 100 × 420 = 192 For the boy of age 15 years : 22 × ? Time = 3 years, Principal = 390300 − x ⇒ = 192 −126 = 66 100 ∴Amount after 3 years compounded annually ⇒ ? = 300 Ans.94.(b) ? % of 150 + 75 × 18 = 1380 b g FGH1 + 100 IJK 3 4 = 390300 − x ......(ii) ? ⇒ × 150 + 1350 = 1380 100 From (i) & (ii), we get 3000 ⇒? = ⇒ ? = 20 150
  • 23. QA – 23 ] [ Practice Paper Ans.95.(b) C.P. of 50 kg. of tea ⇒ 35 K = 315 ⇒ K = 9 = Rs. (20 × 30 + 30 × 25) = Rs. 1350. Hence the numbers are 45 and 63. S.P. of 50 kg. of tea = Rs. (50 × 22.50) Their product is 2835. = Rs. 1125. X 3 Y 5 Z 2 Ans.103.(c) = , = , = Y 4 Z 6 W 3 Loss = Rs. (1350 −1125) = Rs. 225. X Y Z W ⇒ = = = 2×3×2 2d 15 20 24 36 Ans.96.(a) = 3+2 5 ⇒ X : W = 15 : 36 = 5 : 12 LM Average Speed 2xy OP Ans.104.(e) 28 − 5 + 5 − (−13) = 41 N = x+y Q Ans.105.(b) 7.007 + 70.7 + 7.007 = ∴10 d = 60 or d = 6. 84.714 Ans.97.(b) Let x gm of water be taken Ans.106.(d) Last three digits must be Then, Gold = 19x gm & Copper = 9x gm. divided by 8. Let 1 gm of gold be mixed with y gm of Ans.107.(d) Let x be the number Copper. 14% of x = 105 ⇒ x = 750. Then, 19x + 9xy = 15x (1 + y) Ans.108.(b) In one minute, FG 2IJ 1 1 1 1 11 1 7 ⇒y = H 3K + + − = − 10 20 30 15 60 15 60 = Ans.98.(a) (1502)2 − (1498)2 of the tank can be filled. = (1502 − 1498) (1502 + 1498) ∴Whole tank will be filled in = 4 × 3000 = 12000. 60 4 3 = 8 minutes. Ans.99.(b) of 480 ÷ 8 + 82 = ? 7 7 5 1 x x 3 1 Ans.109.(c) × 3 = 192 ∴? = 5 × 480 × 8 + 64 ⇒ x = 24 = 36 + 64 = 100 Ans.110.(d) Given expression Ans.100.(a) 64 ÷ 8 ÷ 4 ÷ 2 0.152 = 8 ÷4 ÷2 = 2 ÷2 = 1 = 0.25 + 0.09 − 0.15 a 2 b 5 Ans.101.(c) = , = 0.152 152 8 b 3 c 7 = = = = 0.8 a b c 0.19 190 10 ⇒ = = 10 15 21 Ans.111.(c) Given expression Ans.102.(d) Let the two numbers be 5K and 7K. 0.01 + 0.08 0.09 0.3 = = = = 100 ∴L C M of 5 K and 7 K = 35 K 0.003 0.003 0.003
  • 24. Practice Paper (Solved) ] [ QA – 24 Ans.112.(a) 11 ∴Remaining of the bucket is filled 20 2 1 3 of of 25.20 = K × of 36 11 3 4 2 by tap B in 15 × = 8 minutes 15 20 ⇒ 4.2 = 54 K seconds. 42 7 Ans.119.(a) A, B and C's shares in the ⇒ K= = 540 90 capital are in the ratio of 12000 × 24 : LM R FG IJ UOP 16000 × 24 : 15000 × 16 , NM S H K VQP 1 5 Ans.113.(c) 1 + 2 3 − 1 + 2 − − i.e. 288 : 384 : 240, T 2 2 W i.e. 18 : 24 : 15, = 1 + 2 [3 − {1 + (2 + 2)}] i.e. 6 : 8 : 5 = 1 + 2 [3 − 5] = 1 + 2 (−2) ∴ Share of C in the profit = 1 − 4 = −3 5 Ans.114.(a) 0.2, (.2)2 = 0.04, = × 45600 = Rs. 12000 19 1 0.2 = 0.222 , 1 ÷ 0.2 = 0.2 = 5 Ans.120.(a) Given expression Ans.115.(a) 5% of (5% of 100) b12.12 + 8.12gb12.12 − 8.12g = 5% of 5 = 1 = 0.25 = b0.25gb0.25 + 19.99g 4 LM8 OP × FH16 IK −15 / 8 8 / 15 1/ 4 +N Q 3 Ans.116.(d) Let x = 6 + 6 + 6+..... LMb128g OP −15/7 −1/5 ⇒x = 2 6+x ⇒x = 6 + x N 3 Q ⇒ x2 − x − 6 = 0 4 8 −1 × 8 = 4 + 1 =4 1 ⇒ (x − 3)(x + 2) = 0 = + 1 2 b g e j 0.25 3 3 128 7 27 7 ⇒x = 3 1 1 1 1 1 − = Ans.117.(d) In one hour, e j e j − 10 15 30 Ans.121.(b) 28 + 10 3 2 − 7−4 3 2 of the cistern will be empty. Re5 + 3 j U − Re2 − U 1 -1 Ans.118.(b) Tap A and Tap B can fill = S T V S W T 2 2 3j V 2 2 W 1 1 2 = e5 + 3 j − e2 − 3 j + = −1 together of the bucket in 12 15 30 one minute. = e5 + 3 j − 1 2+ 3 × 9 2− 3 2+ 3 In three minutes, of the bucket is 20 filled by taps A and B. e j e = 5+ 3 − 2+ 3 = 3 j